Sie sind auf Seite 1von 46

Surgery PACKRAT An 18-year-old patient has a tibia/fibula fracture following a motorcycle crash.

Twelve hours later the patient presents with increased pain despite adequate doses of analgesics and immobilization. Which of the following is the most likely diagnosis? A. avascular necrosis B. myositis ossificans C. compartment syndrome D. reflex sympathetic dystrophy EXPLANATIONS: (u) A. Avascular necrosis occurs primarily in muscles post-traumatically and may not arise for several months after an injury. (u) B. Myositis ossificans is a late complication of fracture resulting from disruption of the blood supply to the bone. (c) C. Compartment syndrome is characterized by a pathological increase of pressure within a closed space and results from edema or bleeding within the compartment. It may occur as an early local complication of fracture. (u) D. Reflex sympathetic dystrophy is characterized by painful wasting of the hand muscles that may be secondary to injury and could occur as a late complication. A patient presents to the Emergency Department with complaints of increasing pain in the right lower extremity. He has a history of a non-displaced proximal tibial fracture treated with application of a long leg cast 2 days prior. On exam there is marked swelling of the toes and the patient refuses to move them secondary to pain. An injection of meperidine fails to relieve the patient's pain. Clinical intervention is needed to prevent what complication? A. osteomyelitis B. traumatic arthritis C. Volkmann's contracture D. malunion of fracture fragments EXPLANATIONS: (u) A. Osteomyelitis results from a source of infection that may include an open fracture, but not a closedfracture. (u) B. Traumatic arthritis only occurs as a result of fractures that involve the articular surface. (c) C. This patient has classic findings of acute compartment syndrome. Volkmann's contracture may occur from an untreated compartment syndrome or an arterial injury. (u) D. Malunion of the fracture fragments occur when there is poor reduction of the fracture fragments. Non-displaced fractures do not require reduction. A 38 year-old male sustained a fracture of the left distal tibia following a 25-foot fall and is taken to the operating room for an open reduction internal fixation of the distal tibia. Sixteen hours post-op, the patient develops sustained pain, which is not relieved with narcotics. On passive range of motion of the toes the patient "yells" in agony. The patient also states that the top of his foot has decreased sensation. On physical examination the physician assistant notes that the leg is swollen and the foot is cool to touch. Based upon this information what diagnostic testing should be done? A. X-ray of the lower leg and ankle. B. Doppler studies. C. Bone scan. D. Compartment pressure

Explanations (u) A. X-rays of the lower leg and ankle will only determine bone placement. (u) B. Doppler studies will confirm the presence of a decreased pulse. (u) C. A bone scan is not indicated in the evaluation of compartment syndrome. (c) D. Compartmental pressures should be obtained as soon as possible. If they are elevated this is a surgical emergency. A 35 year-old male placed in a thumb spica cast for a scaphoid fracture presents complaining of forearm and hand pain that is not relieved with pain medication and elevation. Which of the following is the earliest physical exam sign for his current condition? A. Slow capillary refill B. Loss of two-point discrimination C. Absent peripheral pulses D. Pain with passive stretch. Explanations (u) A. Capillary refill is not affected early in compartment syndrome. (c) B. Loss of two-point discrimination can be the earliest sign of compartment syndrome. (u) C. Peripheral pulses are poor indicators of compartment syndrome as they remain intact until late. (a) D. Pain with passive stretch is a subjective finding early and must be differentiated from pain of the original injury. Although a reliable finding it may be difficult to reproduce in the cast. 25 year-old male presents to the ED with left calf pain and cramping, as well as nausea and vomiting. He admits to partying with cocaine all night. He describes his urine as a dark brown color. Serum creatine kinase (CK) is 1325 IU/L (Normal Range 32-267 IU/L). Which of the following is the initial mainstay of therapy for this condition? A. IV rehydration B. Fasciotomy C. Toradol (Ketorlac) D. Hydrotherapy Explanations (c) A. IV rehydration with crystalloids for 24 to 72 hours is the mainstay of therapy for rhabdomyolysis. (u) B. Fasciotomy is indicated for compartment syndrome. (h) C. NSAIDS, such as Toradol (Ketorlac), should not be used due to the vasoconstrictive effects on the kidneys. (u) D. Hydrotherapy is not useful or indicated for Rhabdomyolysis A 42 year-old male presents complaining of a sudden onset of a severe intermittent pain originating in the flank and radiating into the right testicle. He also complains of nausea and vomiting. On examination the patient is afebrile, but restless. Examination of the abdomen reveals tenderness to palpation along the right flank with no rebound or direct testicular tenderness. Urinalysis reveals a pH of 5.4 and microscopic hematuria, but is otherwise unremarkable. Which of the following is the most likely diagnosis? A. Bladder cancer B. Nephrolithiasis C. Acute appendicitis D. Acute epididymitis Explanations

(u) A. While bladder cancer may have associated microscopic hematuria, it presents with painless hematuria orirritative voiding symptoms. (c) B. A sudden onset of severe colicky flank pain associated with nausea and vomiting as well as the absence of rebound or direct testicular tenderness makes nephrolithiasis the most likely diagnosis. This is further supported by the presence of hematuria on the urinalysis. (u) C. While an acute abdomen, such as acute appendicitis, is in the differential diagnosis, the absence of fever and peritoneal signs makes this diagnosis less likely. (u) D. The absence of fever as well as non-tenderness to palpation of the testes suggests a renal rather than gonadal cause of the patients symptoms. Which of the following pathophysiological processes is believed to initiate acute appendicitis? A. Obstruction B. Perforation C. Hemorrhage D. Vascular compromise Explanations (c) A. Obstruction of the appendiceal lumen by lymphoid hyperplasia, a fecalith or foreign body initiates most cases of appendicitis. (u) B. See A for explanation. (u) C. See A for explanation. (u) D. See A for explanation. What is the term for blue discoloration about the umbilicus? A. Cullen's sign B. Murphy's sign C. Rovsing's sign D. Turner sign Explanations (c) A. Cullen's sign is a blue discoloration about the umbilicus and can occur in hemorrhagic pancreatitis and results from hemoperitoneum. (u) B. Murphy's sign is seen in liver and gallbladder disease by which the patient abruptly halts deep inspiration due to pain in the right upper quadrant while the examiner's hand is keeping stable pressure to the area. (u) C. Rovsing's sign is positive when the patient experiences right sided abdominal pain with palpation to the left side. This is generally seen in those with appendicitis. (u) D. Turner sign is also seen in severe, acute pancreatitis but is represented by a green-brown discoloration of bilateral flanks A patient presents with abdominal pain in the right lower quadrant, examination reveals increased pain in the right lower quadrant on deep palpation of the left lower quadrant. This commonly known as which of the following? A. Psoas sign B. Murphy's sign C. Rovsing's sign D. Obturator sign Explanations (u) A. Psoas sign is right lower quadrant pain with right leg extension.

(u) B. Murphy's sign is seen in liver and gallbladder disease in which the patient abruptly halts deep inspiration due to discomfort as the examiners hand applies pressure to the right upper quadrant. (c) C. A positive Rovsing's sign can be elicited in a patient with appendicitis when increased pain occurs in the right lower quadrant upon palpation of the left lower quadrant. (u) D. Obturator sign is right lower quadrant pain with internal rotation of the hip. A 25 year-old female presents with right lower quadrant pain, right flank pain, nausea, and vomiting. Her temperature is 39.6 degrees C. There is right CVA tenderness and RLQ tenderness. Pelvic exam is unremarkable. Urinalysis reveals pH 7.0, trace protein, negative glucose, negative ketones, positive blood, and positive nitrates. Specific gravity is 1.022. Microscopic shows 102 RBCs/HPF, 50-75 WBCs/HPF, rare epithelial cells, and WBC casts. The most likely diagnosis is A. acute salpingitis. B. nephrolithiasis. C. acute pyelonephritis. D. appendicitis. Explanations (u) A. Acute salpingitis would be suggested if pelvic exam abnormalities were present. (u) B. Nephrolithiasis does not usually present with fever or casts. Urinalysis will have RBCs present. (c) C. Acute pyelonephritis presents with flank pain, fever, and generalized muscle tenderness. Urinalysis shows pyuria with leukocyte casts. (u) D. This scenario is consistent with acute pyelonephritis, not acute appendicitis. A 26 year-old gravida 0 sexually active female presents to the emergency room complaining of colicky pain in her lower abdomen for the past 12 hours. She passed out earlier in the day while trying to have a bowel movement. Her last menstrual period was 6 weeks ago. She has noted vaginal spotting over the last 24 hours. Vital signs show Temp 37 degrees C, BP 96/60mmHg, P 110, R 16, Oxygen Sat. 98%. Abdominal exam is positive for distension and tenderness. Bowel sounds are decreased. Pelvic exam shows cervical motion and adnexal tenderness. Which of the following is the most likely diagnosis? A. Ectopic pregnancy B. Appendicitis C. Crohn's disease D. Pelvic inflammatory disease Explanations (c) A. High suspicion for ectopic pregnancy should be maintained when any possible pregnant woman presents with vaginal bleeding or abdominal pain. (u) B. Appendicitis presents with nausea, vomiting and periumbilical pain that moves to the right lower quadrant of the abdomen. (u) C. Crohn's disease is more common in women and may present with an acute abdomen. However, pelvic examination would be normal. (u) D. In pelvic inflammatory disease the temperature is usually above 38 degrees C and pelvic pain usually follows onset of cessation of menses A 22 year-old male presents to the clinic complaining of scrotal pain that radiates into the groin. Patient admits to being a weightlifter and was lifting 24 hours prior to this pain developing into the scrotum. The patient admits to being sexually active with only his male partner. Examination reveals a reddened scrotum and it is difficult to distinguish the epididymis from the testes on the right side. Elevation of the right testicle brings relief of the pain. This is known as a positive A. Prehn's sign.

B. Cullen's sign. C. Rovsing's sign. D. Murphy's sign. Explanations (c) A. Prehn's sign is seen in epididymitis when elevation of the scrotum with the affected epididymis to the level of the symphysis pubis brings relief from the pain. (u) B. Cullen's sign is a bluish discoloration of the umbilicus resulting from hematoperitoneum. (u) C. Rovsing's sign is seen in appendicitis when pressure on the left quadrant produces pain in the right lower quadrant. (u) D. Murphy's sign is seen in acute cholecystitis with a sharp increase in tenderness when the gall bladder touches the examining hand causing a stop of the inspiratory effort. Which of the following conditions is most suggestive of an asymptomatic abdominal aortic aneurysm? A. abdominal mass B. hypertension C. chest pain D. syncope EXPLANATIONS: (c) A. Symptomatic abdominal aortic aneurysm presents with pulsating upper abdominal mass. (u) B. Hypertension is not suggestive of symptomatic abdominal aortic aneurysm. (u) C. Abdominal aortic aneurysm presents with mid-abdominal or lower back pain. (u) D. Syncope is not common in abdominal aortic aneurysm, unless it ruptures. A 12-year-old boy presents to the office with pain in his legs with activity gradually becoming worse over the past month. He is unable to ride a bicycle with his friends due to the pain in his legs. Examination of the heart reveals an ejection click and accentuation of the second heart sound. Femoral pulses are weak and delayed compared to the brachial pulses. Blood pressure obtained in both arms is elevated. Chest xray reveals rib notching. Which of the following is the most likely diagnosis? A. abdominal aortic aneurysm B. pheochromocytoma C. coarctation of the aorta D. thoracic outlet syndrome EXPLANATIONS: (u) A. Abdominal aortic aneurysm is usually asymptomatic until the patient has dissection or rupture. It is uncommon in a child. (u) B. Pheochromocytoma classically causes paroxysms of hypertension due to catecholamine release from the adrenal medulla, but does not cause variations in blood pressure in the upper and lower extremities. (c) C. Coarctation is a discrete or long segment of narrowing adjacent to the left subclavian artery. As a result of the coarctation, systemic collaterals develop. X-ray findings occur from the dilated and pulsatile intercostal arteries and the "3"is due to the coarctation site with proximal and distal dilations. (u) D. Thoracic outlet syndrome occurs when the brachial plexus, subclavian artery, or subclavian vein becomes compressed in the region of the thoracic outlet. It is the most common cause of acute arterial occlusion in the upper extremity of adults under 40 years old. A 19 year-old female presents with complaint of palpitations. On examination you note the patient to have particularly long arms and fingers and a pectus excavatum. She has a history of joint dislocation

and a recent ophthalmologic examination revealed ectopic lentis. Which of the following echocardiogram findings would be most consistent with this patient's physical features? A. right atrial enlargement B. aortic root dilation C. pulmonic stenosis D. ventricular septal defect Explanations (u) A. Patients with Marfan's syndrome commonly have mitral valve prolapse and possibly aortic regurgitation (high frequency diastolic murmur at the third right intercostal space). Right atrial enlargement, pulmonic stenosis and ventricular septal defect are not commonly seen. (c) B. This patient has the signs and symptoms consistent with Marfan's syndrome. Ectopia lentis, aortic root dilation and aortic dissection are major criteria for the diagnosis of the disease. (u) C. See A for explanation. (u) D. See A for explanation A 56 year-old male presents to the office with a history of abdominal aortic aneurysm. He was told that he will need On going evaluation to assess whether the aneurysm is expanding. What is the recommended study to utilize in this situation? A. plain film of the abdomen B. serial abdominal exam C. ultrasound of the abdomen D. angiography of the abdominal aorta Explanations (u) A. Although some abdominal aortic aneurysms are calcified, abdominal radiography may demonstrate the calcified outline of the aneurysm. However, about 25% of aneurysms are not calcified and cannot be visualized by plain x-ray. (u) B. Serial abdominal exams are not sensitive in detecting progression of abdominal aortic aneurysms. (c) C. An abdominal ultrasound can delineate the transverse and longitudinal dimensions of an abdominal aortic aneurysm and may detect mural thrombus. Abdominal ultrasound is best used to screen patients at risk for the development of this condition. (u) D. Contrast aortography is used commonly for the evaluation of patients with aneurysms before surgery, but it has no role in the serial assessment of patients being followed on a chronic basis. Which of the following is a proven risk factor for the development of abdominal aortic aneurysm? A. Infective endocarditis B. Diabetes mellitus C. Cigarette smoking D. Alcohol abuse Explanations (u) A. Infective endocarditis is not associated with the development of abdominal aortic aneurysm. (u) B. Diabetic patients do have a higher rate of atherosclerosis, but there is no clear causal evidence of diabetics being at higher risk for the development of abdominal aortic aneurysm. (c) C. Cigarette smoking is the primary risk factor for the development of aortic aneurysms. (u) D. Alcohol abuse is not related to development of abdominal aortic aneurysm. Postmenopausal patient is diagnosed with grade I breast cancer. The tumor is 0.7 cm in size, estrogen-receptor positive, and axillary nodes are negative. After undergoing a lumpectomy, which of the following adjuvant therapy is indicated for this patient? A. chemotherapy

B. tamoxifen C. ovarian ablation D. bisphosphonate therapy EXPLANATIONS: (u) A. Chemotherapy is indicated in breast cancer with tumors larger than 1 cm in site. (c) B. Tamoxifen is the adjuvant therapy of choice in post menopausal estrogen receptor positive axillary node negative breast cancer. (u) C. While ovarian ablation is a form of hormonal therapy, it is not the initial treatment of choice. (u) D. Bisphosphonate therapy is used in breast cancer patients with metastasis disease for the bone. A 28 year-old female with diabetes mellitus type 2 sustains a partial thickness burn to her left upper arm and her chest when hot grease spilled on her at home. The burn to her arm is circumferential and the estimated total body surface burned is 18%. She has no allergies. The most appropriate treatment of this patient would include A. outpatient application of silver sulfadiazine. B. debridement of all intact blisters. C. IV cefazolin (Ancef, Kefzol). D. transfer to a burn center. Explanations (u) A. Although treatment may include silver sulfadiazine dressings, this patient should not be treated as an outpatient. Also see D for explanation. (u) B. Debridement of intact blisters remains controversial, however many authorities recommend leaving intact blisters intact and only debride ruptured blisters. (u) C. If used in the care of a burn patient, the antibiotic selected should have activity against Pseudomonas and S. aureus. Cefazolin does not have any antipseudomonal activity. (c) D. Reasons for transfer to a burn center include a partial thickness burn covering greater than 10% of total body surface area. In addition, burns in patients with pre-existing medical conditions, such as diabetes, that could complicate their management, prolong recovery, or affect their outcome, is also a reason for transfer to a burn center. A 72 year-old female is being evaluated for recurrent kidney stones. PE reveals no abnormal findings. Laboratory findings show elevated calcium and decreased phosphate levels. Which of the following is the most likely diagnosis? A. Pheochromocytoma B. Adrenal insufficiency C. Hyperparathyroidism D. Breast cancer (u) A. Pheochromocytoma may lead to hypercalcemia but the patient does not have any signs or symptoms suggestive of pheochromocytoma, such as hypertension, headache, profuse sweating,or weight loss. (u) B. Adrenal insufficiency, Addison's disease, would reveal, in addition to the hypercalcemia,anorexia, nausea and vomiting, weight loss, and cutaneous hyperpigmentation, none of which areevident in this patient. (c) C. The majority of patients with hyperparathyroidism are asymptomatic. Recurrent nephrolithiasis may be one of the presentations of primary hyperparathyroidism. Measurement of parathyroid levels would be the initial laboratory test for the evaluation of hypercalcemia. (a) D. Hypercalcemia may be the earliest manifestation of a malignancy and this must be investigated. Most often the signs and symptoms of a malignancy will cause the patient to seek medical care.

Malignancy is the second leading cause of hypercalcemia, behind hyperparathyroidism. Nulliparity is a risk factor for breast cancer and the most common presenting sign in a breast mass. A 56 year-old female four days post myocardial infarction presents with a new murmur. On examination the murmur is a grade 3/6 pansystolic murmur radiating to the axilla. She is dyspenic at rest and has rales throughout all her lung fields. Blood pressure is 108/68 mmHg, pulse 70 bpm. Which of the following would be the definitive clinical intervention? A. Intra-aortic balloon counterpulsation B. Mitral valve replacement C. Coronary artery bypass surgery D. Immediate fluid bolus Explanations (u) A. Although part of the primary treatment to reduce mitral regurgitation, it is not definitive. (c) B. MVR is the definitive intervention to correct MR caused by papillary muscle rupture. (u) C. CABG may be necessary if significant blockage is found, but it will not correct the mitral regurgitation. (u) D. A fluid bolus is indicated if the patient is hypotensive. 28 year-old male presents with burns sustained from hot grease splashed on his left hand earlier this afternoon. The burn extends from his palm to the volar aspect of his wrist and has an erythematous base, covered by an intact blister. There are a few small scattered blisters over the dorsum of the left hand. Which of the following is the initial intervention of choice? A. Tetanus prophylaxis B. Admission to a burn unit C. Intravenous fluid administration D. Debridement of blisters Explanations (c) A. Tetanus prophylaxis should be initially considered in all burn patients. (u) B. Admission to a burn unit is not indicated for adult patients with uncomplicated partial thickness burns covering less than 15 to 20% of total body surface area (TBSA). (u) C. IV fluids are indicated for severe partial thickness burns covering more than 10% TBSA or in burns with complications. (u) D. Debridement of blisters is controversial, however blisters on the palms and soles should remain intact. Which of the following hyperthyroid patients would be the best candidate for radioiodine therapy? A. A 30-year-old patient with toxic adenoma. B. A 50-year-old man with subacute thyroiditis. C. A patient over age 65 with Grave's disease. D. A pregnant woman with Hashimoto's thyroiditis. EXPLANATIONS: (u) A. Patients with toxic solitary nodules may be treated with surgery or radioactive iodine. Surgery isrecommended for patients under 40 years of age. (u) B. Subacute thyroiditis is usually self-limited. Thyroid iodine uptake is low in this condition, thus rendering radioactive iodine ineffective. (c) C. Radioactive iodine is the recommended treatment for overactive thyroid tissue in patients without risk for subsequent thyroid cancer, leukemia, or other malignancies.

(h) D. Hyperthyroid states are well tolerated during pregnancy. Usual treatment would be small doses of propylthiouracil, which does cross the placenta. Rarely, fetal hypothyroidism may occur; however, this is of much less consequence than using radioiodine therapy A 40-year-old male is hit in the face with a baseball. There is nasal deformity with bleeding. The most appropriate initial management is to A. treat the hematoma with I&D and antibiotics. B. consult with an ENT for immediate reconstructive nasal surgery. C. reduce septal defect using open technique. D. maintain nasal patency and nasal cosmesis. EXPLANATIONS: (u) A. Septal hematoma is less likely due to the finding of epistaxis, and is not of highest priority. (u) B. Reconstructive nasal surgery is a delayed procedure. (u) C. Open reduction is not indicated for nasal trauma. (c) D. Maintain nasal patency until closed reduction can be attempted in 1 week. Which of the following is the selected method for the prevention of venous thromboembolism in a 38year-old male undergoing an inguinal hernia repair? A. early ambulation B. elastic stockings C. intermittent pneumatic compression D. low-molecular weight heparin EXPLANATIONS: (c) A. Early ambulation is recommended for prophylaxis of venous thromboembolism in low-risk, minor procedures when the patient is under 40 years of age and there are no clinical risk factors. (u) B. Elastic stockings are indicated for patients at moderate risk of venous thromboembolism in ages 40-60 with minor procedures with additional thrombosis risk factor, or major operations for patients under age 40 without additional clinical risk factors. (u) C. Intermittent pneumatic compression is indicated in patients undergoing a major operation plus an increased risk of bleeding. (u) D. Low molecular weight heparin is indicated in patients undergoing orthopedic surgery, neurosurgery, or trauma with an identifiable risk factor for thromboembolism. A 16-year-old male presents with increasing pain and swelling of his right scrotum. The right testicle is extremely tender to palpation on examination. A Doppler ultrasound demonstrates decreased blood flow. Which of the following is the most appropriate intervention? A. oral doxycycline B. emergent surgery C. incision and drainage D. scrotal elevation and ice packs EXPLANATIONS: (h) A. Oral doxycycline is the treatment of choice for epididymitis. While epididymitis and testicular torsion present similarly, the Doppler ultrasound in epididymitis would show increased blood flow, not decreased. (c) B. Once a diagnosis of testicular torsion is suspected, emergent surgery is indicated to have the bestpossible chance of salvaging the testicle (85-97% chance if less than 6 hours). Any other treatment measures delay the definitive treatment and increase the risk of testicular ischemia and infarction. (h) C. Incision and drainage is indicated for treatment of abscesses, not testicular torsion.

(h) D. Scrotal elevation and ice packs are indicated for adjunct treatment of epididymitis, not testicular torsion. Which of the following is the most appropriate intervention for a stage I testicular seminoma? A. Watchful waiting B. Chemotherapy initially C. Orchiectomy and radiation D. Orchiectomy and chemotherapy Explanations (u) A. Surveillance is an option in stage I disease of a nonseminoma testicular tumor. (u) B. Patients with stage IIC and stage III are treated with chemotherapy. (c) C. Inguinal orchiectomy followed by retroperitoneal radiation therapy cures about 98% of patients with stage I seminoma. (u) D. Chemotherapy is used for later stage tumors (II/III) and followed by surgery in stage III tumors Which of the following beta-adrenergic blocking agents has cardioselectivity for primarily blocking beta1 receptors? A. Propranolol (Inderal) B. Timolol (Blocadren) C. Metoprolol (Lopressor) D. Pindolol (Visken) Explanations (u) A. Propranolol and timolol are nonselective beta-adrenergic antagonists. (u) B. See A for explanation. (c) C. Metoprolol is selective for beta-1 antagonists (u) D. Pindolol is an antagonist with partial agonist activity. A 55-year-old male presents complaining of episodic substernal chest pain that occurs especially during strenuous exercise. Suspecting coronary artery disease (CAD), an exercise stress test is ordered. The test is considered to be abnormal if which of the following occurs? A. Systolic blood pressure increases during exercise. B. The heart rate reaches maximal value during exercise. C. Random premature ventricular beats occur at peak exercise. D. A 2 mm ST-segment depression is seen on the ECG at peak exercise. Explanations (u) A. An increase in systolic blood pressure is an expected normal response during this test. (u) B. Heart rate should reach maximal value during exercise. (u) C. Random premature ventricular beats are not uncommon during peak exercise and do not indicate CAD. (c) D. A 2 mm ST-segment depression is suggestive of cardiac ischemia and is considered to be an abnormal finding. The treatment of choice for a patient with WPW (Wolff-Parkinson-White Syndrome) who has recurrent episodes of supraventricular tachycardia is which of the following? A. IV Verapamil (Calan, Isoptin) B. Digoxin (Lanoxin) C. Surgical ablation of the accessory pathway D. Radiofrequency ablation of bypass tracts

Explanations (h) A. Caution should be employed when using digitalis or intravenous verapamil in patients with the WPW syndrome and atrial fibrillation, since these drugs can shorten the refractory period of the accessory pathway and can increase the ventricular rate, thereby placing the patient at increased risk for ventricular fibrillation. (h) B. See A for explanation. (u) C. Although surgical ablation is an effective therapy, surgery has been virtually replaced by the advent of radiofrequency catheter ablation. Surgery should be reserved for patients who do not respond to other therapy. (c) D. Catheter ablation of bypass tracts is possible in more than 90% of patients and is the treatment of choice in patients with symptomatic arrhythmias. It is safer, more cost-effective, and just as successful as surgery. A 60 year-old female presents for pre-op evaluation for surgical treatment of a tri-malleolar fracture of the leftankle. Which of the following puts her at an increased risk for infection post surgery? A. Recent URI B. Diabetes mellitus C. Heart murmur D. Swelling of the ankle Explanations (u) A. Recent URI, heart murmur and swelling of the ankle would not put her at risk for an infection post surgery. (c) B. Diabetes mellitus puts her at risk for infection post surgery. (u) C. See A for explanation.(u) D. See A for explanation A patient with renal artery stenosis is unresponsive to medical therapy. Which of the following is the nextmost appropriate intervention? A. Stenting of the renal artery B. Nephrectomy C. Radioactive iodine D. Lifestyle modifications Explanations (c) A. Stenting of the renal artery will dilated the narrowed area interfering with blood supply to the kidney. (u) B. Saving the kidney and not removing the kidney is indicated. (u) C. Radioactive iodine is indicated for multinodular goiter and not for renal artery stenosis. (u) D. Lifestyle modifications in patients who are unresponsive to medical therapy prior to stenting will not change thenatural course of the disease. This is recommended following stenting or surgery. Patient with chronic rheumatoid arthritis on maintenance prednisone and methotrexate undergoes surgery. She develops hyponatremia, hypoglycemia, and hypotension. In addition to IV fluid therapy, which of the following is the best initial therapy? A. Epinephrine B. Fludrocortisone (Florinef) C. Hydrocortisone D. Glucagon Explanations (u) A. The shock that results from adrenal crisis is not responsive to vasopressors.

(u) B. Fludrocortisone may be utilized in treating the convalescent phase, but not acute phase, of adrenal crisis. (c) C. The acute phase of adrenal crisis is treated with IV saline and hydrocortisone. (u) D. See C for explanation. Which of the following is the most common indication for operative intervention in patients with chronic pancreatitis? A.Weight loss B. Intractable pain C. Exocrine deficiency D. To decrease risk of cancer Explanations (u) A. While weight loss is common with chronic pancreatitis, it is not an indication for surgical intervention. (c) B. Indications for surgical treatment of chronic pancreatitis include severe pain that limits the patient's functioning or intractable pain despite the use of non-narcotic analgesics and absence of alcohol intake. (u) C. While the majority of patients go on to develop diabetes mellitus 25 years after the clinical onset of chronic pancreatitis, this is not an indication for surgical intervention as it would lead to more severe exocrine deficiency. (u) D. While the possible presence of pancreatic cancer is an indication for surgery, there is no indication for prophylactic surgery to decrease the risk of cancer A patient with prostate cancer has a nonpalpable, focal lesion, and the patient is reluctant to have surgery at this time. Which of the following would best monitor disease progression? A. Periodic rectal exams B. Transrectal ultrasonography C. Measurements of serum acid phosphatase D. Measurements of prostate-specific antigen Explanations (u) A. Many prostate carcinomas are contained within the gland, making it difficult to assess progression with a digital examination alone. (u) B. Ultrasonography is used largely for staging disease, not monitoring disease progression. (u) C. Serum acid phosphatase is more predictive of metastatic disease than PSA measurement, but its use haslargely been replaced by PSA. (c) D. PSA measurement correlates well with volume and stage of disease and is the recommended examination formonitoring disease progression. 52 year-old male with history of hypertension and hyperlipidemia presents with an acute myocardial infarction. Urgent cardiac catheterization is performed and shows a 90% occlusion of the left anterior descending artery. The other arteries have minimal disease. Ejection fraction is 45%. Which of the following is the treatment of choice in this patient? A. Coronary artery bypass grafting (CABG) B. Streptokinase C. Percutaneous coronary intervention (PCI) D. Warfarin (Coumadin) Explanations

(u) A. Percutaneous coronary intervention is a better, less invasive alternative to CABG for single vessel coronary artery disease. (h) B. Streptokinase is not commonly used for treatment of acute myocardial infarction because it is ineffective atopening the occluded artery and reducing mortality. Streptokinase would be harmful because it would increasethe risk of bleeding. (c) C. Immediate coronary angiography and primary percutaneous coronary intervention have been shown to be superior to thrombolysis. (u) D. Warfarin is used to prevent thrombosis and not for acute treatment. 78 year-old male with history of coronary artery disease status post CABG and ischemic cardiomyopathy presents with complaint of progressive dyspnea and orthopnea. He also complains of lower extremity edema. The patient denies fever, chest pain, or cough. On physical examination, vital signs are BP 120/68, HR 75 and regular, RR 22, afebrile. You note the patient to have an S3 heart sound, jugular venous distention, and 2+ lower extremity edema. The patient is admitted and treated. Upon discharge from the hospital, the patient should be educated to monitor which of the following at home? A. Daily weights B. Daily spirometry C. Daily blood glucose D. Daily fat intake Explanations (c) A. Home monitoring of daily weights can alert the health care provider to the early recognition of worsening heart failure. (u) B. Spirometry monitoring is important in a patient with asthma, not heart failure. (u) C. Daily blood glucose monitoring is important in a patient with diabetes, not heart failure. (u) D. Daily fat intake is important, but will not improve his heart failure management. A 59 year-old otherwise healthy female develops acute dyspnea and chest pain one week post total abdominal hysterectomy. Echocardiogram demonstrates normal heart size with normal right and left ventricular function. Lung scan demonstrates two segmental perfusion defects. Which of the following is the next step in the management of this patient? A. Anticoagulation B. Embolectomy C. Thrombolysis D. Inferior vena cava filter Explanations (c) A. Anticoagulation is the treatment of choice in patients with pulmonary embolism with normal ventricular function and no absolute contraindications. (u) B. Embolectomy is not indicated as initial treatment of a pulmonary embolism in patients with normal ventricular function. (h) C. Thrombolysis is contraindicated in patients within 10 days of having major surgery. (u) D. An inferior vena cava filter is considered in patients with contraindications to anticoagulation therapy or failed anticoagulation therapy. Lab results for a post-operative oliguric patient reveals an increased BUN to creatinine ratio. The patient has a low fractional excretion of sodium (less than 1%). Which of the following is the most likely diagnosis? A. prerenal azotemia B. acute tubular necrosis

C. acute glomerulonephritis D. obstructive uropathy Explanations (c) A. Patients who have prerenal azotemia with otherwise normal kidneys will have severe sodium retention in order to help to save fluid. The amount of sodium in the urine is therefore very low. (u) B. Acute tubular necrosis may occur in the post-operative setting but these kidneys are damaged and unable to save sodium. (u) C. Acute glomerulonephritis is a complication of a streptococcal infection wherein the immune complexes damagethe glomeruli and lead to hematuria, red blood cell casts, and proteinuria. (u) D. Although patients who undergo abdominal surgery are at risk for damage to the genital urinary system, thesepatients will not have sodium retention because it is a post renal, not a prerenal injury. A 60 year-old patient returned from the recovery room to the floor following a subtotal gastrectomy. At 3 AM the next morning, the patient's temperature is 102 F (39 C) and pulse is 112/min. Which of the following is the most likely cause? A. wound infection B. atelectasis C. phlebitis D. shock Explanations (u) A. Wound infection does not present this early. (c) B. Atelectasis is the most common pulmonary complication, affecting 25% of patients with abdominal surgery. It is more common in elderly and overweight patients and occurs within the first 12 to 24 hours postoperatively. (u) C. Phlebitis occurs more commonly after the second postoperative day. (u) D. In shock, the pulse is usually thready and the temperature is not elevated A 54 year-old female who has diabetes presents with rubor, absence of hair, and brittle nails of her left foot. She complains of leg pain that awakens her at night. Examination reveals a femoral bruit with diminished popliteal and pedal pulses on the left side. The most appropriate therapy would be A. vasodilator therapy. B. bypass surgery. C. exercise program. D. embolectomy. Explanations (u) A. Vasodilator therapy is not indicated. (c) B. Bypass surgery is indicated in the presence of rest pain and provides relief of symptoms in 80 to 90% of patients. (u) C. While an exercise program is appropriate with claudication, rest pain is a surgical indication. (u) D. Embolectomy is used for acute arterial occlusion The first step in the treatment of a patient with an intestinal obstruction and no comorbid diseases is A. nasogastric decompression. B. invasive hemodynamic monitoring. C. abdominal exploration. D. administration of antibiotics. Explanations

(c) A. Nasogastric decompression is indicated in all but mild cases of obstruction to prevent distal passage of swallowed air and minimize distension. (u) B. Invasive hemodynamic monitoring is needed only if the patient has underlying cardiac, pulmonary, or renal disease. (u) C. Abdominal exploration is considered only if the obstruction does not resolve in 24 to 48 hours or if peritoneal findings, fever, or rapidly progressing abdominal pain occur. (u) D. Antibiotics are given only if surgery is to be done. Which of the following treatments will most benefit the diabetic patient with two vessel coronary disease? A. Stent placement B. Percutaneous balloon angioplasty C. Medical management D. Coronary artery bypass graft Explanations (u) A. See D for explanation. (u) B. See D for explanation. (u) C. See D for explanation. (c) D. CABG is the treatment of choice in a diabetic with two or three vessel disease. Patient presents to the emergency department with right upper quadrant pain over eight hours, nausea, and vomiting. On exam there is a fever of 101.2 degrees F. Ultrasound shows a distended gallbladder. What is the most appropriate management of this patient? A. Oral analgesics B. Diagnostic peritoneal lavage C. Proton pump inhibitors D. Laparoscopic cholecystectomy Explanations (u) A. See D for explanation. (h) B. Diagnostic peritoneal lavage is used to detect intraabdominal bleeding from trauma and not to treat acute cholecystitis and may delay appropriate treatment. (u) C. Proton pump inhibitors are used to treat GERD or PUD. (c) D. The proper treatment for acute cholecystitis is IV fluids, antibiotics, pain control, and surgery. Cholecystectomy is the definitive treatment for acute cholecystitis and laparoscopic cholecystectomy is the procedure of choice. A 57 year-old male presents with acute bilateral lower extremity weakness and urinary incontinence that began after he fell earlier today. His examination is significant for bilateral lower extremity sensory deficits and weakness along with decreased rectal sphincter tone. Which of the following is the most appropriate intervention? A. Epidural steroids B. Oral NSAIDs C. Physical therapy D. Surgery Explanations (h) A. While epidural steroids can be effective in treating lumbar disc herniation, in the case of cauda equine syndrome, immediate surgical decompression is mandatory.

(h) B. NSAIDs may be beneficial in some cases of lumbar muscle strain and disc herniation. They are not appropriate for management of cauda equina syndrome, immediate surgical decompression is mandatory. (h) C. Physical therapy may be beneficial in some cases of lumbar muscle strain and disc herniation but it is not appropriate for management of cauda equina syndrome. (c) D. Cauda equina syndrome is a rare but serious surgical emergency because the duration of nerve compression is inversely correlated with the likelihood of full neurologic recovery A patient with a history of severe peptic ulcer disease is 5 weeks status post Billroth I surgery. One week ago he restarted his normal diet and has had the onset of severe nausea, abdominal cramping, and lightheadedness that occur approximately thirty minutes after eating. The abdominal exam reveals a healing surgical scar without areas of unusual tenderness or any palpable masses. Which of the following is the most likely diagnosis? A. Anxiety disorder B. Celiac sprue C. Dumping syndrome D. Irritable bowel syndrome Explanations (u) A. Anxiety disorders can cause a wide variety of somatic syndromes such as those mentioned, but generally not with such a straightforward dietary trigger. (u) B. Celiac sprue can cause similar symptoms as those listed, can develop at any age and can be worsened by the ingestion of gluten containing products. The patient's recent surgery makes dumping syndrome a much greater possibility. (c) C. Dumping syndrome typically occurs after Billroth type I surgeries as well as gastric bypass surgeries when the patient attempts to eat a large amount of simple sugars. (u) D. Irritable bowel syndrome is a diagnosis of exclusion and is associated with alternation in bowel habits. In patents with diabetic retinopathy, what clinical intervention is most successful in preserving vision? A. Panretinal laser photocoagulation B. Iridectomy C. Radial keratotomy D. Vitrectomy Explanations (c) A. Panretinal laser photocoagulation is indicated for preservation of vision in patients with diabetic retinopathy. (h) B. Iridectomy is of no value in preserving the retina and iridectomy is harmful in this situation due to the trauma it causes to the eye. (h) C. Radial keratotomy is indicated to correct myopia. This surgery destroys normal eye architecture and has no benefit in diabetic retinopathy. (h) D. Vitrectomy is indicated for treatment of retinal tears and not to preserve an intact retina. A 20 year-old presents 30 minutes after being struck by a hockey puck in the mouth. On physical examination a central incisor is missing from its socket. The patient has the tooth wrapped in tissue paper and the root appears intact. Which of the following is the most appropriate next step in the treatment of this patient? A. Administration of IM penicillin

B. No treatment is warranted C. Place tooth in saline and refer to plastic surgery for reimplantation D. Immediately reimplant the tooth and refer to an oral surgeon Explanations (u) A. Penicillin is not indicated for treatment of an avulsed tooth. (h) B. Reimplantation is warranted as this is a permanent tooth with root intact. Primary teeth are never reimplanted. (u) C. See D for explanation. (c) D. Avulsed permanent teeth should be cleansed, transported in Hanks solution or saline and reimplanted by an oral surgeon within one hour. A 62 year-old male presents with a right hilar mass. Needle-biopsy of the mass reveals the presence of small-cell carcinoma and a bone scan reveals the presence of scattered hot spots throughout the skeleton. Which of the following is the most appropriate treatment? A. Lobectomy B. Pneumonectomy C. Thoracic radiation therapy D. Combination chemotherapy Explanations (u) A. Small-cell carcinoma of the lung is rarely treatable with surgical resection. Surgery may be indicated as part of the treatment protocol for small peripheral lesions without any evidence of metastasis. (u) B. See A for explanation. (u) C. While thoracic radiation therapy has been shown to be beneficial for patients with limited smallcell lung cancer, no benefit has been observed for patients with extensive disease defined as the presence of metastatic disease. (c) D. Combination chemotherapy is the treatment of choice for a patient with small-cell carcinoma of the lung. A 60 year-old male has unstable angina, but is otherwise healthy. A 90% lesion is found in the left main coronary artery. Which of the following interventions is most appropriate? A. Thrombolysis with t-PA B. Medical management with nitrates C. Coronary artery bypass graft (CABG) D. Percutaneous transluminal coronary angioplasty Explanations (u) A. Thrombolysis is recommended in acute embolic occlusion, not chronic. (u) B. Medical management is appropriate only for patients who are not surgical candidates. (c) C. CABG is indicated in patients with stenosis of the left main coronary artery and those with threevessel CAD (u) D. Percutaneous transluminal coronary angioplasty is not the management of choice in left mainstem artery disease because of increased potential complications and mortality. Patient with adrenal insufficiency is taking hydrocortisone 25 mg daily. What should the patient do with the hydrocortisone dose when they develop a minor illness such as a cold? A. Stop the hydrocortisone until the illness resolves. B. Continue the current dose that the patient is taking. C. Increase the dose to 50 mg daily until the illness resolves.

D. Increase the dose to 250 mg daily until the illness resolves Explanations (h) A. Stopping the hydrocortisone would cause adrenal crisis. (u) B. See C for explanation. (c) C. To better mimic the normal physiologic response the baseline dose should be doubled for the duration of the illness. Doses should be increased 5-10 fold with major events such as surgery. (u) D. See C for explanation 45 year-old type 2 diabetic female with history of cholelithiasis presents to the clinic with 2-3 episodes of sudden, severe epigastric pain that radiates to her shoulder. She has associated nausea and vomiting. Temperature is 101 degrees F and she is experiencing chills. Today her eyes appear yellow in color. Which of the following is the most likely diagnosis for this patient? A. Postcholecystectomy syndrome B. Cholangitis C. Gastroesophageal reflux disease D. Pancreatic cancer Explanations (u) A. The patient has no history of previous gall bladder surgery. (c) B. Cholangitis is characterized by a history of biliary pain, fever, chills, and jaundice associated with episodes of abdominal pain. (u) C. Gastroesophageal reflux disease (GERD) is characterized by heartburn. Fever and jaundice are not typical features of GERD. (u) D. Pancreatic cancer, although a possibility, is characterized by chronic weight loss, epigastric pain radiating to the back, and occasional jaundice. Fever and chills are not typical features A 52-year-old female presents with diffuse abdominal pain accompanied by distention and visible peristalsis. Ausculation reveals hyperactive bowel sounds. Percussion is tympanic throughout. Palpation reveals mild diffuse tenderness without masses. The most likely diagnosis is A. intra-abdominal abscess. B. intestinal obstruction. C. paralytic ileus. D. cholecystitis. EXPLANATIONS: (u) A. An intra-abdominal abscess would be accompanied by fever and localized tenderness. (c) B. Intestinal obstruction without complications is suggested by crampy pain, abdominal distention, hyperactive bowel sounds, visible peristalsis, and minimal tenderness. (u) C. Bowel sounds in paralytic ileus are absent. (u) D. Cholecystitis is accompanied by localized pain and tenderness Which of the following is the most consistent physical examination finding in a patient with duodenal ulcer? A. Flank tenderness B. Right upper quadrant tenderness C. Epigastric tenderness D. Rebound tenderness Explanations (u) A. Flank tenderness is caused by urologic disorders such as pyelonephritis and renal lithiasis. (u) B. Right upper quadrant tenderness on palpation is a typical feature for cholecystitis.

(c) C. Epigastric tenderness is a key feature of duodenal ulcer. (u) D. Rebound tenderness is a feature of peritonitis from rupture of a hollow viscus and is not seen with just the presence of duodenal ulcer. A 29 year-old female G4P2Ab1 at 20 weeks gestation complains of nausea and vomiting with tenderness in the RUQ. Vital signs reveal the patient to be febrile. On physical examination, the abdominal examination reveals positive bowel sounds in all quadrants with a positive Murphy's sign. Fundus can be palpated at the level of the umbilicus. The skin is warm and dry with slight tenting. Oral mucosa is dry as well. What is the most likely diagnosis? A. Peptic ulcer disease B. Hyperemesis gravidarum C. Cholecystitis D. Viral gastroenteritis Explanations (u) A. Peptic ulcer disease is less frequently seen during pregnancy secondary to increased mucus production and decreased gastric motility. (u) B. Hyperemesis gravidarum usually resolves by the 16th week of pregnancy and is not accompanied by a positive Murphy's sign. (c) C. Gallbladder disease represents one of the most common medical and surgical conditions seen during pregnancy. This is thought to be due to a decrease in gallbladder contractility and lithogenicity of the bile. There is an increased risk in multiparous women. (u) D. Viral gastroenteritis may present with these symptoms, however, viral gastroenteritis does not produce a positive Murphys sign. 70 year old man complains of abdominal pain of four hours duration. He states that he has vomited twice since the onset of pain. He also complains of three days of constipation. He is afebrile and the physical examination is noteworthy for a distended, diffusely tender abdomen with normoactive bowel sounds. His rectal exam reveals hemoccult positive brown stool. Medications include omeprazole (Prilosec) for GERD, digoxin and warfarin (Coumadin) for atrial fibrillation, OTC multivitamins and stool softeners. The abdominal and chest x-rays show no abnormalities. Which of the following is the most likely diagnosis? A. Acute cholecystitis B. Mesenteric infarction C. Perforated duodenal ulcer D. Small bowel obstruction Explanations (u) A. Acute cholecystitis causes right upper quadrant pain with radiation into the shoulder and does not cause hemoccult positive stools. (c) B. Acute onset of severe diffuse abdominal pain in a person with atrial fibrillation warrants the suspicion of mesenteric infarction. Vomiting and constipation may be seen, along with occult blood in the stool. Bowel sounds may be normal. (u) C. Abdominal and chest x-rays would reveal free air under the diaphragm in a ruptured duodenal ulcer and a small bowel obstruction would reveal air-fluid levels with distended bowel loops. (u) D. See C for explanation A 62 year-old male presents with complaints of vague epigastric abdominal pain associated with jaundice and generalized pruritus. Physical examination reveals jaundice and a palpable non-tender gallbladder, but is otherwise unremarkable. Which of the following is the most likely diagnosis?

A. Viral hepatitis B. Pancreatic cancer C. Acute cholecystitis D. Gilbert's syndrome Explanations (u) A. While viral hepatitis may cause jaundice, the liver is enlarged and tender. (c) B. Pancreatic cancer is suggested by the vague epigastric pain with the jaundice resulting from biliary obstruction due to cancer involving the pancreatic head. The presence of a palpable non-tender gallbladder (Courvoisier's sign) also indicates obstruction due to the cancer. (u) C. While acute cholecystitis may present with jaundice and an enlarged gallbladder, the pain is classically colicky and located in the right upper quadrant. On physical examination with deep inspiration and palpation of the right subcostal area increased pain and respiratory arrest (Murphy's sign) is usually seen. (u) D. Gilbert's syndrome is the most common of the hereditary hyperbilirubinemias. It is most often diagnosed near puberty or adult life based on results of a comprehensive metabolic panel. A 40 year-old female complains of acute right upper quadrant pain radiating to the back and low grade fever. Laboratory evaluation indicates the presence of urinary bilirubin and an elevation of serum alkaline phosphatase. Which of the following is the most likely diagnosis? A. cholecystitis B. viral hepatitis C. Gilbert's syndrome D. Dubin-Johnson syndrome Explanations (c) A. The presence of urinary bilirubin indicating conjugated hyperbilirubinemia coupled with the elevation of serum alkaline phosphatase suggests biliary obstruction that may lead to cholecystitis. (u) B. Although viral hepatitis presents with conjugated hyperbilirubinemia, aminotransferase elevation would predominate, not alkaline phosphatase. (u) C. Gilbert's syndrome presents as unconjugated hyperbilirubinemia so urinary bilirubin would be absent. (u) D. Although this hereditary disorder presents with conjugated hyperbilirubinemia, liver enzymes would not be elevated 50 year-old male with history of alcohol abuse presents with acute, severe epigastric pain radiating to the back. The patient admits to an episode of coffee ground emesis. On examination he is ill-appearing with a rigid, quiet abdomen and rebound tenderness. Which of the following is the most likely diagnosis? A. Abdominal aortic aneurysm B. Perforated duodenal ulcer C. Acute myocardial infarction D. Cholecystitis Explanations (u) A. A patient with an abdominal aortic aneurysm may present with pain radiating to the back, however would not have coffee ground emesis or an acute abdomen. (c) B. Perforation of a duodenal ulcer causes sudden, severe pain, with rebound tenderness and rigid abdomen on physical examination. It is often associated with coffee ground emesis. (u) C. A patient with an acute myocardial infarction may have pain radiating to the back, however would not have hematemesis or an acute abdomen.

(u) D. Cholecystitis presents with right upper quadrant pain and is not typically associated with coffee ground emesis or rebound tenderness. A 65 year-old homeless male with a history of pancreatitis is seen in the emergency department for vomiting, upper abdominal pain, back pain and weakness. He is cachetic, pale and jaundiced. A 4-5 cm mass is palpable in the mid to right hypochondrium. What is the most likely diagnosis? A. Chronic cholecystitis B. Carcinoma of head of pancreas C. Fibrolipoma D. Primary biliary cirrhosis Explanations (u) A. Chronic cholecystitis is not typically associated with weight loss or cachexia. There would not be a palpable mass. (c) B. Seventy-five percent of pancreatic cancers are in the head. Risk factors include age, tobacco use, obesity, chronic pancreatitis, family history and previous abdominal radiation. (u) C. Fibrolipoma may present as an abdominal mass, but would not cause weight loss and illness. (u) D. Primary biliary cirrhosis most commonly presents with generalized urticaria and is not associated with an abdominal mass A 16-year-old male is found on physical examination to have a history of cryptorchidism of the right testes. This was surgically corrected at age 18 months. This patient should be monitored for the development of A. prostatic cancer. B. testicular cancer. C. bladder cancer. D. penile cancer. EXPLANATIONS: (u) A. See B for explanation, African americans are at a higher risk for prostate cancer. (c) B. Cryptorchidism increases the risk of testicular cancer. In fact, the major predisposing risk factor is cryptorchidism unrepaired until after age two and Caucasian men at highest risk. (u) C. See B for explanation. (u) D. See B for explanation. A 60-year-old male presents with a recent history of change of bowel habits, weight loss, and blood and mucus in his stools. The most likely diagnosis is A. hemorrhoids. B. colorectal carcinoma. C. acute diverticulitis. D. fistula-in-ano. EXPLANATIONS: (u) A. Hemorrhoids may present with blood noted in the stool, but is not accompanied by weight loss. (c) B. Colorectal cancer presents with weight loss, change of bowel habits, and blood in stool. (u) C. Acute diverticulitis presents with abdominal pain and tenderness, fever, and peritoneal findings. (u) D. Patients with fistula-in-ano presents with severe pain, and there may be blood in the stool, but weight loss is not seen. A 76 year-old female presents to the ED with the worst abdominal pain in her life. The pain began following a large meal and is located periumbilically. Although she is writhing in pain, she does not have an exacerbation of the pain on palpation of the abdomen. She has a history of coronary artery disease, asthma, and atrial fibrillation. Which of the following is the most likely diagnosis?

A. Toxic megacolon B. Mesenteric thrombosis C. Fulminant hepatitis D. Acute diverticulitis with perforation Explanations (u) A. Toxic megacolon is a complication seen with ulcerative colitis or electrolyte abnormalities in which the bowel loses its tone. (c) B. This patient is at risk for mesenteric ischemia due to advanced age, atherosclerosis and atrial fibrillation. This is the classic presentation for this condition with pain out of proportion to physical examination findings. (u) C. Fulminant hepatitis is most likely to cause malaise, loss of taste, lethargy, and right upper quadrant pain. (u) D. Acute diverticulitis with perforation will cause left lower quadrant abdominal pain and severe pain on palpation due to the peritonitis that occurs from the perforation of bowel contents. Ulcerative colitis usually presents with which of the following? A. bloody diarrhea B. toxic megacolon C. fever and left quadrant pain D. alternating constipation and diarrhea Explanations (c) A. Ulcerative colitis typically presents with episodic bloody diarrhea, lower abdominal cramps, and urgency to defecate. (u) B. Toxic megacolon is a complication of ulcerative colitis, but it is not a common presentation. (u) C. Left lower quadrant pain and a palpable mass accompanied by fever is classic for diverticulitis. (u) D. Irritable bowel syndrome usually presents with constipation, painless diarrhea with mucous, or alternating constipation and diarrhea. Which of the following is the most common type of skin cancer? A. basal cell B. melanoma C. atypical nevi D. squamous cell Explanations (c) A. Basal cell cancer is the most common cause of skin cancer usually occurring on sun-exposed areas. (u) B. While malignant melanoma is the leading cause of death from skin disease, it is not the most common skin cancer. (u) C. Atypical nevi are associated with melanoma. They are diagnosed clinically, not histologically. Any atypical nevi suspected to be melanomas should be removed. (u) D. Squamous cell carcinomas also occurs in sun-exposed areas, but are less frequent than basal cell cancers. A 72 year-old man presents with acute left lower quadrant abdominal pain. He has nausea, vomiting, and constipation. He has a fever of 101 F and guarding and rebound tenderness in his left lower quadrant. His white blood cell count is elevated. He has no prior history of gastrointestinal disease. Which of the following is the most likely diagnosis? A. Inflammatory bowel disease B. Irritable bowel syndrome

C. Viral gastroenteritis D. Acute diverticulitis Explanations (u) A. Inflammatory bowel disease typically presents in a younger population. (u) B. Irritable bowel syndrome is not associated with nausea, vomiting and fever. It usually presents in a younger population. (u) C. Viral gastroenteritis typically does not localize to the left lower quadrant. (c) D. Acute abdominal pain, fever, left lower abdominal tenderness, and leukocytosis are hallmark signs of acute diverticulitis. A 62 year-old male is brought to the emergency department with acute hematemesis. The patient denies a previous history of vomiting. His wife states he has chronic liver disease. Physical examination reveals a distended abdomen without rebound, guarding or organomegaly. There is a fluid wave. Which of the following is the most likelydiagnosis? A. Esophageal varices B. Mallory-Weiss tear C. Arteriovenous malformation D. Perforated duodenal ulcer Explanations (c) A. Esophageal varices are dilated submucosal veins that develop in a patient with underlying portal hypertension. The most common cause of portal hypertension is cirrhosis. (u) B. A patient with a Mallory-Weiss tear would have a history of retching but would not have a distended abdomen. (u) C. Most arteriovenous malformations are asymptomatic. If symptomatic they would have symptoms of a slow bleed. (u) D. A patient with perforated duodenal ulcer would have rebound and guarding on examination An 18 month-old female presents to the Emergency Department having possibly swallowed a hearing aid battery within the past hour. She is drooling and appears anxious but parents have noticed no stridor or dyspnea. She has no history of previous esophageal injury. Physical examination is unremarkable. Chest radiograph reveals a radiopaque round object at the distal esophagus. Which of the following is the most appropriate treatment option? A. Observation for 24 hours B. Esophagoscopy for removal C. Barium swallow D. Bronchoscopy Explanations (h) A. Batteries must be removed as they can induce mucosal injuries in as little as one hour of contact time. (c) B. Esophagoscopy is the procedure of choice for acutely ingested foreign bodies. (u) C. A barium swallow is a diagnostic option but will not provide treatment. (u) D. Bronchoscopy would be the procedure of choice for an airway foreign body, not esophageal. A patient diagnosed with Barrett's esophagus is at an increased risk for the development of what type of cancer? A. Squamous cell B. Transitional cell C. Adenocarcinoma

D. Atypical carcinoid Explanations (u) A. Squamous cell is not typical for esophageal cancer. (u) B. Transitional cell is a cancer of the bladder. (c) C. The most serious complication of Barrett's esophagus is esophageal adenocarcinoma. (u) D. Atypical carcinoid is more typical of lung cancer A 41 year-old female presents to you for medical screening advice. Her 44 year-old sister passed away recently 18 months after diagnosis of metastatic colon cancer. Which of the following is the most appropriate advice for this patient? A. Double contrast barium enema now and repeat every 5 years if normal B. Rectal occult blood testing annually until age 50 then sigmoidoscopy every 3 years C. Rectal occult blood testing annually until age 50 then screening colonoscopy every 5 years D. Screening colonoscopy now and repeat every 3-5 years if normal Explanations (u) A. See D for explanation. (u) B. See D for explanation. (u) C. See D for explanation. (c) D. Hereditary factors are believed to contribute to up to 30% of colorectal cancers. Relative risk is 3.8 times if the family member's cancer was diagnosed at less than 45 years of age. Recommended screening in a single first degree relative with colorectal cancer diagnosed before age 60 is beginning colonoscopy at age 40 or ten years younger than age at diagnosis of youngest affected first-degree relative. Then if negative, every 5 years Which of the following is the most appropriate study for diagnosing Hirschsprung disease? A. Rectal biopsy B. Stool leukocyte test C. CT of the abdomen and pelvis D. Fecal occult blood test Explanations (c) A. A rectal biopsy showing the absence of ganglion cells in both the submucosal and muscular layers of the involved bowel is the most appropriate diagnostic study for Hirschsprung disease. (u) B. Stool leukocyte testing can indicate an infectious etiology of diarrhea and is not indicated in the diagnosis of Hirschsprung disease. (u) C. Radiographic examination may show dilated proximal colon and absence of gas in the pelvic colon, but is not diagnostic for Hirschsprung disease. (u) D. Fecal occult blood testing is not indicated in the diagnosis of Hirschsprung disease What is the pathologic mechanism of Hirschsprung's disease? A. Pyloric circular muscle hypertrophy causing gastric outlet obstruction B. Absence of ganglion cells in the mucosal and muscular layers of the colon C. A defect in the diaphragm leading to protrusion of the abdominal viscera into the thoracic cavity D. Absence of relaxation of the lower esophageal sphincter and lack of peristalsis in the esophageal body Explanations (u) A. Pyloric circular muscle hypertrophy causing gastric outlet obstruction describes pyloric stenosis. (c) B. Hirschsprung's disease results from an absence of ganglion cells in the mucosal and muscular layers of the colon.

(u) C. A defect in the diaphragm leading to protrusion of the abdominal viscera into the thoracic cavity describes congenital diaphragmatic hernia. (u) D. Absence of relaxation of the lower esophageal sphincter and lack of peristalsis in the esophageal body describes achalasia of the esophagus A person presenting with bleeding esophageal varicies should be treated with which of the following while awaiting arrival of endoscopy? A. Carafate (Sucralfate) B. Octreotide (Sandostatin) C. Omeprazole (Prilosec) D. Enoxaparin (Lovenox) Explanations (u) A. Carafate is not indicated in acute gastrointestinal bleeding. (c) B. Octreotide is a vasoacctive drug used in the treatment of GI bleeding as well as somatostatin, vasopressin, and terlipressin. Somatostatin and octreotide are preferred due to safety and less incidence of serious side effects. (u) C. Omeprazole, a proton pump inhibitor, is not indicated in acute gastrointestinal hemorrhage. (h) D. Enoxaparin will increase bleeding and therefore contraindicated in GI bleeding A pregnant female presents at 32 weeks gestation with painless rectal bleeding and a bulging perianal mass when straining which goes away. Which of the following is the most appropriate management of this patient? A. Hemorrhoidectomy B. Metronidazole (Flagyl) C. Psyllium (Metamucil) D. Sclerotherapy Explanations (u) A. Hemorrhoidectomy and sclerotherapy are reserved for severe Grade III and IV hemorrhoids. (u) B. Metronidazole is not indicated in the treatment of hemorrhoids. (c) C. Dietary fiber or psyllium bulk laxatives can be used to decrease straining with defecation. (u) D. See A for explanation The main complication with the use of transjugular intrahepatic portosystemic shunt (TIPS) procedure is which of the following? A. increased portal pressures resulting in further esophageal varices B. increased portal pressures resulting in a worsening of cirrhosis C. Budd-Chiari syndrome D. increased risk of encephalopathy Explanations (u) A. TIPS procedures are performed in order to shunt blood away from the liver parenchyma which in essence lowers portal pressures lessening the risk for esophageal varices. (u) B. TIPS procedures, since they cause of bypass of the liver parenchyma, result in a lessening of the blood flow into the liver which does not cause cirrhosis to progress. (u) C. Budd-Chiari syndrome is a thrombosis of the hepatic vein. It is not a common complication of the TIPS procedure. (c) D. TIPS procedures involve the placement of a stent in the liver in order to shunt blood away from the portal vein into the hepatic vein which bypasses the cirrhotic liver parenchyma. Its main

complication is encephalopathy from the accumulation of toxic substances in the brain since the liver no longer acts as a filter. A 65 year-old patient with known history of alcohol and tobacco abuse presents with solid food dysphagia. The patient also has a 24 lb weight loss over the past 6 months. Which of the following is the most appropriate intervention? A. endoscopy with biopsy B. chest x-ray C. barium esophagogram D. CT scan of the thorax Explanations (c) A. Endoscopy with biopsy establishes the diagnosis of esophageal cancer with a high degree of reliability. (u) B. Chest x-ray may show adenopathy, a pulmonary or bony metastases or sign of tracheoesophageal fistula. Barium esophagogram is obtained as the first study to evaluate the dysphagia. CT scan should be obtained once the diagnosis of carcinoma has been made to evaluate for pulmonary or hepatic metastases, lymphadenopathy, and local tumor extension. None of these tests will reveal the diagnosis of carcinoma. (u) C. See B for explanation. (u) D. See B for explanation. A 65 year-old patient with adenocarcinoma of the colon in remission presents for follow-up. Which of the following tumor markers should be monitored? A. AFP B. CEA C. CA 19-9 D. CA-125 Explanations (u) A. AFP is used to monitor recurrence of hepatocellular carcinoma. (c) B. CEA is used to monitor recurrence of colon carcinoma. (u) C. CA 19-9 is used to monitor recurrence of pancreatic carcinoma. (u) D. CA-125 is used to monitor recurrence of ovarian carcinoma. A 42 year-old patient who is being treated for colon cancer with chemotherapy develops nausea and vomiting. Which of the following drugs would be the most effective in controlling the nausea and vomiting? A. scopolamine (Scopace) B. meclizine (Antivert) C. ondansetron (Zofran) D. loperamide (Imodium) Explanations (u) A. Scopolamine and meclizine are effective against motion sickness, but ineffective against substances that act directly on the chemoreceptor trigger zone. (u) B. See A for explanation. (c) C. Ondansetron selectively blocks 5-HT3 receptors in the periphery (visceral afferent fibers) and in the brain (chemoreceptor trigger zone). It is indicated for use in chemotherapy induced nausea and vomiting. (u) D. Loperamide has low anti-emetic potency.

A patient presents complaining of vague anal discomfort. On examination, the patient is noted to have a few small external hemorrhoids and edema in the anal region. Which of the following is the most appropriate intervention? A. proctoscopy followed by a hemorrhoidectomy B. increased dietary fiber and sitz baths C. hemorrhoidal banding D. inject a sclerosing agent Explanations (u) A. Hemorrhoidectomy should be used for permanently prolapsed internal hemorrhoids. (c) B. Most hemorrhoids respond well to conservative treatment such as fiber and sitz baths. (u) C. Banding and injection of sclerosing agents are used if mild prolapse, enlargement, or intermittent bleeding is present. (u) D. See C for explanation A 42 year-old male with a history of constipation presents with complaints of severe pain with defecation described as feeling like he is "tearing apart." He has also noted occasional small amounts of blood on toilet paper. External examination of the rectum is unremarkable and an internal rectal exam cannot be performed due to severe pain when attempted. Which of the following is the most likely diagnosis? A. Proctitis B. Anal fissure C. Rectal prolapse D. Internal hemorrhoids Explanations (u) A. Proctitis is usually caused by anorectal infections that produce symptoms of anorectal discomfort, tenesmus, constipation, and discharge from the rectum. (c) B. Anal fissures are easily diagnosed from history alone with the classic finding of severe pain upon defecation. Constipation is also a common cause of the trauma that leads to development of a fissure. (u) C. Rectal prolapse is commonly seen in elderly females with complaints that include an anal mass, rectal bleeding and a change in bowel habits. (u) D. While internal hemorrhoids may cause rectal bleeding, tearing pain is an uncommon complaint unless there is evidence of thrombosis of irreducible tissue. A patient presents complaining of periumbilical pain. Which of the following anatomical sites is this finding associated with? A. Bladder B. Stomach C. Pancreas D. Small bowel Explanations (u) A. Pain from the bladder, uterus, or colon causes hypogastric pain. (u) B. Pain from the stomach, duodenum, or pancreas causes epigastric pain. (u) C. See B for explanation. (c) D. Pain from the small intestine, appendix, or proximal colon causes periumbilical pain 25 year-old female presents to the ED with an open fracture of the left fibula sustained from an auto accident. The patient has no neurological findings. In addition to stabilization of the patient which of the following should be immediately initiated?

A. Antibiotic therapy B. Apply a bi-valve cast C. Reduce the fracture D. Surgical debridement Explanations (c) A. Antibiotic therapy should be started immediately along with tetanus if needed. (h) B. Putting a bi-valve cast on immediately is not indicated and could be harmful. (h) C. Reducing the fracture in the absence of neurological symptoms could cause nerve injury and is not indicated immediately. (u) D. Surgically debriding the injury should be initiated immediately after antibiotics are begun. Congenital absence of ganglionic nerve cells innervating the bowel wall is seen in which of the following conditions? A. Hirschsprung's disease B. Meckel's diverticulum C. Chagas disease D. Hashimoto's hypothyroidism Explanations (c) A. Hirschsprung disease, also termed congenital aganglionic megacolon, results from a lack of ganglion cells in the bowel wall. (u) B. Meckel's diverticulum results from the failure of the vitelline duct to separate from the intestine during early gestation leading to an outpouching in the intestinal tract. (u) C. Chagas disease results from infection with the Trypanosoma parasite. Chronic Chagas disease may lead to the loss of dorsal motor nuclear cells of the vagus nerve, resulting in megaesophagus and megacolon. (u) D. Ineffective peristalsis of the colon from hypothyroidism may result in constipation but not Hirschsprung disease Which of the following laboratory markers are helpful in establishing the diagnosis of ovarian cancer in a 55 year-old post-menopausal women? A. Ca-125 B. CEA C. LDH D. AFP Explanations (c) A. Ca-125 is one of the best tumor markers in epithelial ovarian cancer. (u) B. CEA (carcinoembryonic antigen) is a tumor marker used for colon cancer. (u) C. Only in young girls and adolescents should an LDH and AFP be ordered as there is a greater likelihood of a malignant germ cell tumor. (u) D. See C for explanation Which of the following is better visualized with transesophageal echocardiogram (TEE) than transthoracic echocardiogram? A. Ventricular wall motion B. Pulmonary arteries C. Right ventricle D. Left atrial appendage Explanations

(u) A. See D for explanation. (u) B. The pulmonary arteries are not well visualized during either procedure. (u) C. See D for explanation. (c) D. TEE allows 2-D and Doppler imaging of the heart through the esophagus. Given the close proximity of the esophagus to the heart, high-resolution images can be obtained, especially of the left atrium, mitral valve apparatus, and aorta Which of the following is the therapy of choice for long-term management of esophageal varices in a patient who cannot tolerate beta blocker therapy? A. Octreotide (Sandostatin) B. Sclerotherapy C. Transjugular intrahepatic portosystemic shunt D. Sengstaken-Blakemore tube Explanations (u) A. Octreotide is used to control acute variceal bleeding but does not have a role in long-term management. (c) B. Sclerotherapy is effective in decreasing the risk for rebleeding in a patient with esophageal varices. (u) C. Transjugular intrahepatic portosystemic shunt (TIPS) decreases rebleeding more than sclerotherapy or ligation but is associated with a higher incidence of encephalopathy and has shown no decrease in mortality. TIPS is used in patients who fail endoscopic or pharmacologic therapy. (u) D. The Sengstaken-Blakemore tube is rarely used today in the treatment of acute variceal bleeding. These are not indicated for long-term use. A 72 year-old male with a new diagnosis of congestive heart failure and atrial fibrillation, develops episodes of hemodynamic compromise secondary to increased ventricular rate. A decision to perform elective cardioversion is made and the patient is anticoagulated with heparin. Which test should be ordered to assess for atrial or ventricular mural thrombi? Answers A. Electrocardiogram B. Chest x-ray C. Transesophageal Echocardiogram D. C-reactive protein Explanations (u) A. Electrical conduction will not assess for mural thrombi. (u) B. A chest x-ray will not visualize the left atria and ventricles to assess for mural thrombi. (c) C. Transesophageal echocardiography allows for determination of mural thrombi that may have resulted from atrial fibrillation. (u) D. C-reactive protein is not going to give you any information regarding thrombi. This test is used to identify the presence of inflammation. A patient with a bowel perforation secondary to a gunshot wound is being prepped for surgery. Appropriate antibiotic prophylaxis and treatment includes which of the following? A. cefoxitin and gentamicin B. vancomycin and penicillin G C. nafcillin, gentamicin, and metronidazole D. metronidazole, clindamycin, and cefoxitin EXPLANATIONS:

(u) A. Cefoxitin will provide reasonably good activity against Bacteroides fragilis, but has poor activity against gram-positive cocci and Enterococci. See C for explanation. (u) B. The spectrum of activity of vancomycin is limited to gram-positive cocci. See C for explanation. (c) C. Nafcillin provides treatment for penicillinase-resistant organisms Gentamicin covers many gram negative aerobes, and metronidazole is effective against a wide variety of anaerobic bacteria. (u) D. Metronidazole, clindamycin, and cefoxitin essentially have the same spectrum of activity. They are effective against gram-negative anaerobes. Primary hyperparathyroidism is characterized by which of the following? A. decreased serum magnesium B. decreased serum phosphate C. increased hematocrit D. increased bone density EXPLANATIONS: (u) A. See B for explanation. (c) B. Parathyroid hormone stimulates the osteoclasts to increase bone resorption, leading to elevated calcium levels. It works in the kidney to increase calcium reabsorption and increase renal excretion of phosphorous. Hematocrit is not affected by parathyroid hormone. (u) C. See B for explanation. (u) D. See B for explanation. A 20-year-old male presents with a mass in the groin. On inspection with the patient standing a symmetric, round swelling is noted at the external ring. When the patient lies down the mass disappears.The patient denies any trauma. The most likely diagnosis is A. an indirect inguinal hernia. B. a direct inguinal hernia. C. an obturator hernia. D. a femoral hernia. EXPLANATIONS: (u) A. An indirect inguinal hernia is typically elliptic that does not reduce easily. (c) B. A direct inguinal hernia is symmetrical, round and disappears easily with the patient lying down. (u) C. An obturator hernia is more commonly seen in elderly women and are rarely palpable in the groin. (u) D. Femoral hernias are rare in males and do not typically reduce with lying down. Which of the following is the selected method for the prevention of venous thromboembolism in a 38year-old male undergoing an inguinal hernia repair? A. early ambulation B. elastic stockings C. intermittent pneumatic compression D. low-molecular weight heparin EXPLANATIONS: (c) A. Early ambulation is recommended for prophylaxis of venous thromboembolism in low-risk, minor procedures when the patient is under 40 years of age and there are no clinical risk factors. (u) B. Elastic stockings are indicated for patients at moderate risk of venous thromboembolism in ages 40-60 with minor procedures with additional thrombosis risk factor, or major operations for patients under age 40 without additional clinical risk factors. (u) C. Intermittent pneumatic compression is indicated in patients undergoing a major operation plus an increased risk of bleeding.

(u) D. Low molecular weight heparin is indicated in patients undergoing orthopedic surgery, neurosurgery, or trauma with an identifiable risk factor for thromboembolism 65 year-old female presents to the office with a six-month history of back pain. The patient states that she is shrinking and thinks she is about an inch shorter than she was a year ago. Serum parathyroid hormone, calcium, phosphorus, and alkaline phosphatase are all normal. Which of the following would you most likely see on the x-ray of her spine? Answers A. Radiolucent lesions B. Demineralization C. Chondrocalcinosis D. Subperiosteal resorption Explanations (u) A. Pagets disease of bone presents with bone pain, kyphosis, bowed tibias, large head, and deafness. The initial lesions are destructive and radiolucent. Pagets disease has a normal serum calcium and phosphate, but the serum alkaline phosphatase is elevated. (c) B. Osteoporosis presents with varying degrees of back pain and loss of height is common. The serum calcium, parathyroid hormone, phosphorus, and alkaline phosphatase are normal. Xray findings demonstrate demineralization in the spine and pelvis. (u) C. Chondrocalcinosis is the presence of calcium-containing salts in articular cartilage and is commonly seen in hyperparathyroidism, diabetes, hypothyroidism, and gout. (u) D. Hyperparathyroidism is frequently asymptomatic. Serum parathyroid hormone and serum calcium are elevated. X-ray findings include demineralization, subperiosteal resorption of bone especially in the radial aspects of the fingers. A 35 year-old female who was a back seat passenger in a vehicle which was involved in a head-on collision is brought to the ED. She is able to tell you that she is having difficulty moving both of her legs and is experiencing bilateral leg pain as well. She is embarrassed because she has "wet myself." Physical examination reveals markedly diminished sensory and motor function of both legs and decreased rectal sphincter tone. Which of the following is the most likely diagnosis? A. Herniated disc at L5-S1 and L4-L5 B. An anterior cord lesion C. Cauda equina syndrome D. An L2 lesion Explanations (u) A. Herniated disc at L5-S1 and L4-L5 will cause a dermatomal sensory change in the legs. (u) B. An anterior cord lesion would cause complete paralysis below the lesion with loss of pain and temperature sensation. (c) C. Cauda equina syndrome is a massive central disc protrusion that causes variable degrees of paralysis. Bowel and bladder function may be impaired with saddle anesthesia. This condition is a surgical emergency. (u) D. An L2 lesion Which of the following is the most consistent physical examination finding associated with a hydrocele? A. Tender and swollen testicle B. Palpable painless mass on the testicle C. Nontender scrotal mass that transilluminates D. Inability to get exam finger above scrotal mass

Explanations (u) A. A tender and swollen testicle is seen in infectious states, such as acute orchitis. (u) B. A palpable painless mass on the testicle is consistent with testicular cancer until proven otherwise. (c) C. A hydrocele is a fluid-filled mass that is nontender to palpation. Diagnosis is readily made by transillumination, however evaluation is still warranted as 10% of testicular tumors may have an associated hydrocele. (u) D. Inability to get exam finger above the scrotal mass is consistent with a hernia and helps to distinguish a hernia from a hydrocele. Trousseau's sign is usually associated with which of the following? A. Hypocalcemia B. Hypokalemia C. Osteomalacia D. Hyperparathyroidism Explanations (c) A. Trousseau's sign is associated with hypocalcemia. (u) B. See A for explanation. (u) C. See A for explanation. (u) D. See A for explanation A 43 year-old male is found to have an elevated serum calcium on routine pre-employment laboratory testing. Further laboratory testing demonstrated: Calcium 11.3 mg/dL (8.8 - 10.2 mg/dL) Ionized Calcium 6.2 mg/dL (4.6 - 5.3 mg/dL) Phosphorus 2.1 mg/dL (3.0 - 4.5 mg/dL) What is the most likely diagnosis? A. Adrenal insufficiency B. Hyperparathyroidism C. Osteoporosis D. Pagets disease Explanations (u) A. See B for explanation. (c) B. The presence of elevated total and ionized calcium are consistent with hyperparathyroidism. (u) C. See B for explanation. (u) D. See B for explanation Which of the following is a cause of prerenal azotemia? A. Infection B. Renal toxins C. Poor renal perfusion D. Urinary tract obstruction Explanations (u) A. Infection is associated with interstitial nephritis, which is considered a cause of intrinsic renal azotemia. (u) B. This is one of the causes of intrinsic renal azotemia. (c) C. Renal hypoperfusion is the cause of prerenal azotemia, which may be rapidly reversible when renal blood flow and glomerular ultrafiltration pressure are restored. (u) D. Urinary tract obstruction is the cause of postrenal azotemia.

A patient presents to the office with worsening fatigue, weight loss, and weakness. She notes that she is having recurrent bouts of abdominal pain and has been losing her pubic hair. Patient is found to have orthostatic hypotension. Which of the following conditions is most likely? A. Cushing's syndrome B. Pheochromocytoma C. Primary hyperparathyroidism D. Addison's disease Explanations (u) A. Cushing's syndrome is caused by an increase in the cortisol levels in the body. These patients will have hypertension, buffalo hump, striae, and proximal muscle weakness. (u) B. Pheochromocytoma is caused by an increase in the release of catecholamines from the adrenal medulla. These patients will have episodic hypertension followed by sustained hypertension and bouts of diaphoresis and shakiness. (u) C. Patients with primary hyperparathyroidism are most likely to be asymptomatic. If these patients have symptoms, it is most likely that they will have abdominal pain, renal stones, and bone pain because of the resultant increase in the serum calcium levels. Patients with hyperparathyroidism will have an increase in the serum calcium level as a result of the parathyroid gland retaining calcium. (c) D. Patients with Addison's disease have primary adrenal failure from an autoimmune problem in the adrenal gland or due to hemorrhage into the adrenal gland. These patients are not able to make glucocorticoids, mineralocorticoids, or sex hormones which result in hypotension, hyperpigmentation (from an increase in the ACTH and MSH hormones) and are hyponatremic. Which of the following is the most common presentation for an elderly female patient with primary hyperparathyroidism? A. Abdominal pain B. Renal lithiasis C. Acute pancreatitis D. Asymptomatic Explanations (u) A. Patients with primary hyperparathyroidism may have increased calcium levels which result in abdominal pain and constipation but this is not the most common manifestation. (u) B. Patients with primary hyperparathyroidism may have renal lithiasis as a result of the hypercalcemia but it is not the most common presentation. (u) C. Patients with acute pancreatitis may have hypocalcemia as a result of the calcium soaps that are formed in order to help to process the food that is eaten. It is not related to hyperparathyroidism; rather it is most commonly caused by gallstones and alcoholism. (c) D. Patients with primary hyperparathyroidism are most commonly found to have this disease by an incidental finding of hypercalcemia on routine laboratory testing as a result of screening. Up to 0.1% of the adult population has this condition which is most commonly seen in females over age 50. What is considered the most common physical examination finding for intestinal obstruction? A. Distention B. Fluid wave C. Rigidity D. Tenderness Explanations

(c) A. Abdominal distention is the most common hallmark of all kinds of intestinal obstructions though its presence can be variable depending on the duration and exact location of the obstruction. (u) B. Fluid wave is noted with ascites, not intestinal obstruction. (u) C. Rigidity is most often minimal to absent except in the most extreme of late presenting strangulated obstructions and is more commonly found early in the course of peritonitis and related phenomenon. (u) D. Tenderness, like rigidity, is not a predominant finding in obstructed bowel until very late in the course of advancing, untreated cases. At this time distention and symptoms of shock will also be present and command the clinical picture. An adult presents with a three month history of progressive severe muscle cramps, extremity paresthesias and lethargy which began shortly after a thyroidectomy for a malignant thyroid lesion. Which of the following is the most likely diagnosis? A. Hypoparathyroidism B. Hypothyroidism C. Hyperparathyroidism D. Hyperthyroidism Explanations (c) A. Hypocalcemia secondary to hypoparathyroidism is commonly seen as a complication of thyroidectomy. (u) B. Hypothyroidism is possible without replacement therapy after thyroidectomy but would not result in the tetanylike symptoms and hypocalcemia. (u) C. Hyperparathyroidism will have a different constellation of symptoms and will be typified by elevated serum calcium. (u) D. Hyperthyroidism is not likely status post thyroidectomy though possible with over-aggressive replacement therapy. Symptoms with low serum calcium are the factors that move away from this being the diagnosis. A 22 year-old patient complains of sudden onset of chest pain accompanied by shortness of breath. The patient appears dyspneic. On examination, the trachea is deviated to the left, breath sounds are faint on the right, and the right chest is hyperresonant to percussion. The preferable treatment for this patient would be A. a tracheostomy. B. insertion of a chest tube with underwater seal, left 2nd intercostal space. C. needle thoracotomy right 2nd intercostal space D. a lung scan for pulmonary embolus and begin heparin sodium (Heparin) therapy. Explanations (u) A. Tracheostomy is indicated for upper airway obstruction, not spontaneous pneumothorax. (h) B. This patient has a tension pneumothorax on the right, and insertion of the chest tube on the left would be life threatening. (c) C. Decreased breath sounds and hyperresonance are noted on the side of the pneumothorax; tracheal deviation to the opposite side indicates development of a tension pneumothorax. Treatment consists of inserting a chest tube on the side of the pneumothorax and connecting to an underwater seal. (u) D. Pulmonary embolus may present with chest pain and dyspnea, but physical examination findings do not include decreased breath sounds with hyperresonance and tracheal deviation Post-infarction syndrome (Dressler's syndrome) occurs after acute myocardial infarction presenting as

A. ventricular aneurysm. B. pericarditis and pleuritis. C. cardiac tamponade. D. pleural effusion and rash. EXPLANATIONS: (u) A. See B for explanation. (c) B. Dressler's syndrome is the occurrence of pericarditis and pleuritis several days to weeks following an MI. (u) C. Pericardial tamponade may result from severe pericardial effusion or hemorrhage into the pericardium, but it is not typically associated with Dressler's syndrome. (u) D. Dressler's syndrome is post-MI pericardial inflammation, not pleural effusion or rash. In suspected subarachnoid hemorrhage with a negative head CT, which of the following studies should be used to help establish the diagnosis of subarachnoid hemorrhage? A. complete blood count B. lipid profile C. lumbar puncture D. electrocardiogram EXPLANATIONS: (u) A. A complete blood count has no diagnostic value in the evaluation of a subarachnoid hemorrhage. (u) B. A lipid profile may reveal elevated cholesterol and triglycerides, and risk factors for Intracerebral hemorrhages, but it is of no diagnostic value. (c). C. Although 95% of subarachnoid hemorrhages show blood on head CT, the remaining do not show evidence of hemorrhaging. A lumbar puncture should then be performed and the fluid examined for red blood cells or xanthochromia. (u) D. An electrocardiogram may show diffuse T wave inversion in the precordial leads in a subarachnoid hemorrhage, but these are not always present. Which of the following is the recommended treatment of a large volume hemothorax? A. perform pleurodesis B. drain the empyema C. administer antibiotics D. remove blood EXPLANATIONS: (u) A. Pleurodesis is a procedure by which an irritant is placed into the pleural space following chest tube drainage and lung re-expansion. (u) B. An empyema is an infection, not blood in the pleural space. (u) C. See D for explanation. (c) D. A hemothorax is defined as bleeding into the pleural space. A large volume hemothorax is treated by immediate insertion of a large bore thoracostomy tube to (1) drain existing blood and clot, (2) quantify the amount of bleeding, (3) reduce the risk of fibrothorax, and (4) permit apposition of the pleural surfaces in an attempt to reduce hemorrhage. Episodes of hemoptysis. Chest x-ray reveals a hilar nodule with hilar adenopathy and mediastinal widening. What is the most likely diagnosis? A. Large cell lung cancer B. Adenocarcinoma

C. Bronchiectasis D. Squamous cell lung cancer Explanations (u) A. Large cell lung cancer typically presents with peripheral masses. (u) B. Adenocarcinoma typically presents with peripheral masses. (u) C. Bronchiectasis presents on chest x-ray with no findings or prominent cystic spaces with or without air-liquid levels. (c) D. Squamous cell cancer is common in patients with a smoking history and presents with a central mass, hilar adenopathy and mediastinal widening Which of the following typical findings would be revealed during a sigmoidoscopy on a patient with Crohn's disease of the intestine? A. Rectal pseudopolyps B. Diffuse ulceration and bleeding C. Sheets of WBCs with inflamed mucosa D. Intermittent longitudinal mucosal ulcers and fissures Explanations (u) A. Rectal pseudopolyps are associated with ulcerative colitis rather than Crohn's. (u) B. Diffuse ulcerations and bleeding are more characteristic of ulcerative colitis than Crohn's disease. (u) C. Sheets of WBCs or "pseudomembranes" can be detected in patients with pseudomembranous colitis. (c) D. Ulcerations tend to be linear with transverse fissures in Crohn's disease. These skip lesions are common with Crohn's disease. Which of the following is the most common radiographic presentation of lung abscess? A. cavitation B. pleural thickening C. hilar mass D. hyperinflation Explanations (c) A. Cavitation is seen with lung abscess or progressive primary tuberculosis. (u) B. Pleural thickening is noted in mesothelioma. (u) C. Hilar and mediastinal abnormalities are common on chest radiography in patients with lung cancer. (u) D. Hyperinflation is the main clinical feature in emphysema. Which of the following forms of lung cancer is associated with the poorest prognosis? A. squamous cell B. adenocarcinoma C. large cell D. small cell Explanations (u) A. See D for explanation. (u) B. See D for explanation. (u) C. See d for explanation. (c) D. Small cell lung cancer is the most common type of lung cancer that is metastatic at the time of discovery, and therefore has the poorest prognosis The most reliable site from which to identify the causative organism in cases of osteomyelitis is the A. base of ulcer.

B. blood. C. sinus tract. D. bone. Explanations (u) A. See C for explanation. (u) B. While blood cultures are indicated in acute cases of osteomyelitis, they are only positive in 25-50% of pediatric hematogenous osteomyelitis and 10% of other forms of bone infection. (u) C. Taking specimens for culture from a sinus tract or the base of an ulcer correlate poorly with organisms infecting the bone. (c) D. Samples from needle aspiration of pus in the bone, or from a bone biopsy, are essential to determine the exact causative agent. A 24-year-old male is initially found to have a single nodule in the right lobe of his thyroid gland. He is clinically and chemically euthyroid. The next step is to A. reassure the patient and reassess yearly. B. recheck in 1-3 months. C. do a fine needle aspiration. D. obtain a CT scan of the neck. EXPLANATIONS: (h) A. See C for explanation. (h) B. See C for explanation. (c) C. The combination of fine needle aspiration and radioisotope scanning of a solitary thyroid nodule provides the best diagnostic yield. Because cold nodules may be cancerous, they are generally referred for surgical removal. It is not reasonable to delay the diagnosis. (u) D. Ultrasound is preferred over MRI or CT scan of the thyroid Renal cell carcinoma most commonly presents with which of te following symptoms or signs? A. hypocalcemia B. inguinal pain C. anemia D. hematuria EXPLANATIONS: (u) A. Renal cell cancer may present with hypercalcemia. (u) B. See D for explanation. (u) C. Renal cell cancer may present with polycythemia, not anemia. (c) D. The most common presenting symptom/sign of renal cell carcinoma is hematuria (approximately 60%). Flank pain or abdominal mass is present in about 30% of new cases A patient with hypovolemic shock would most likely exhibit which of the following signs? A. increased CVP; decreased BP; increased pulse rate B. decreased CVP; decreased BP; increased pulse rate C. increased CVP; increased BP; decreased pulse rate D. decreased CVP; increased BP; decreased pulse rate EXPLANATIONS: A. See B for explanation. B. Hypovolemic shock is a condition with a decrease in the amount of circulating blood volume in the intravascular system. A decrease in the amount of circulating volume will result in a decrease in the CVP pressure which is an indirect measurement of the amount of blood in the right ventricle. Less blood in

the vascular system means decreased blood pressure. Since there is less blood in the circulation, the body will attempt to compensate for this by increasing the number of contractions (pulse rate) and the force of those contractions due to increased sympathetic stimulation. (u) C. See B for explanation. (u) D. See B for explanation Which of the following is the pathophysiologic process of a transudative pleural effusion? A. increased fluid production due to increased hydrostatic pressure B. decreased lymphatic clearance of fluid from the pleural space C. infection in the pleural space D. bleeding into the pleural space EXPLANATIONS: (c) A. A transudative pleural effusion occurs in the setting of normal capillary integrity and suggests the absence of local pleural disease. Chronic heart failure accounts for 90% of transudates. Hypoalbuminemia, cirrhosis, and acute atelectasis are also causes of a transudate. (u) B. An exudative pleural effusion results from pleural disease associated with increased capillary permeability or reduced lymphatic drainage. Bacterial pneumonia and cancer are the most common causes of an exudative pleural effusion. (u) C. An empyema is defined as infection in the pleural space that is a form of an exudative pleural effusion. (u) D. A hemothorax is defined as bleeding into the pleural space that is a form of an exudative pleural effusion. 6-year-old child with sickle cell anemia presents with fever and pain over the right tibia. There is tenderness along the anterior tibia, but no pain with motion of the knee or ankle. Which of the following is the most appropriate initial treatment? A. aspirin B. codeine C. cefotaxime D. norfloxacin EXPLANATIONS: (u) A. Aspirin is not indicated for the treatment of osteomyelitis. (u) B. Codeine might relieve pain, but will not treat the cause of osteomyelitis. (c) C. Cefotaxime provides coverage for osteomyelitis caused by staphylococcus or salmonella. (u) D. Norfloxacin is contraindicated in children. The most accurate method of diagnosing thrombophlebitis of the lower leg is A. impedance plethysmography. B. physical exam findings. C. Doppler ultrasound. D. venography. EXPLANATION: (u) A. See D for explanation. (u) B. See D for explanation. (u) C. See D for explanation. (c) D. While impedance plethysmography, physical exam findings, and Doppler ultrasound are useful in diagnosing thrombophlebitis, venography is the most accurate method for diagnosis in the lower leg. The most effective preventive strategy to prevent recurrence of renal lithiasis is which of the following? A. increase in hydration B. early treatment of urinary tract infection

C. limitation of calcium intake D. use of probenecid EXPLANATIONS: (c) A. Keeping the urine dilute is the most effective strategy to prevent crystal accumulation in the urine and the development of urinary stones. (u) B. Treating UTI only affects the formation of calcium pyrophosphate or struvite stones. (u) C. Limitation of calcium in the diet reduces the ability of calcium to bind oxalate leading to the production of calcium oxalate stones. (u) D. Probenecid is a uricosuric agent that promoted uric acid secretion in the urine that may promote more uric acid stones. On physical examination you note diminished breath sounds over the right lower lobe with decreased tactile fremitus and dullness to percussion. Which of the following is the most likely cause? Answers A. asthma B. consolidation C. pneumothorax D. pleural effusion Explanations (u) A. Asthma is characterized by decreased tactile fremitus, but would have resonant to hyperresonant percussion, not dullness. (u) B. Consolidation from pneumonia is characterized by dullness to percussion, but would have an increased, not decreased, tactile fremitus. (u) C. A pneumothorax is characterized by decreased to absent tactile fremitus, but would have a hyperresonant percussion note, not dullness. (c) D. A decreased tactile fremitus and dullness to percussion would be found in a pleural effusion. A 60 year-old male is brought to the ED complaining of severe onset of chest pain and intrascapular pain. The patient states that the pain feels as though "something is ripping and tearing". The patient appears shocky; the skin is cool and clammy. The patient has an impaired sensorium. Physical examination reveals a loud diastolic murmur and variation in blood pressure between the right and left arm. Based upon this presentation what is the most likely diagnosis? Answers A. Aortic dissection B. Acute myocardial infarction C. Cardiac tamponade D. Pulmonary embolism Explanations (c) A. The scenario presented here is typical of an ascending aortic dissection. In an acute myocardial infarction the pain builds up gradually. Cardiac tamponade may occur with a dissection into the pericardial space; syncope is usually seen with this occurrence. Pulmonary embolism is usually associated with dyspnea along with chest pain. (u) B. See A for explanation. (u) C. See A for explanation. (u) D. See A for explanation A 56 year-old male with a known history of polycythemia suddenly complains of pain and paresthesia in the left leg. Physical examination reveals the left leg is cool to the touch and the toes are cyanotic. The popliteal pulse is absent by palpation and Doppler. The femoral pulse is absent by palpation but weak

with Doppler. The right leg and upper extremities has 2+/4+ pulses throughout. Given these findings what is the most likely diagnosis? Answers A. Venous thrombosis B. Arterial thrombosis C. Thromboangiitis obliterans D. Thrombophlebitis Explanations (u) A. See B for explanation. (c) B. Arterial thrombosis has occurred and is evidenced by the loss of the popliteal and dorsalis pedis pulse. This is a surgical emergency. Venous occlusion and thrombophlebitis do not result in loss of arterial pulse. (u) C. See B for explanation. (u) D. See B for explanation. A 22 year-old male received a stab wound in the chest an hour ago. The diagnosis of pericardial tamponade is strongly supported by the presence of A. pulmonary edema. B. wide pulse pressure. C. distended neck veins. D. an early diastolic murmur. Explanations (u) A. Pulmonary edema may result with low output states as seen with myocardial contusions, but it is not strongly suggestive of tamponade. (u) B. Wide pulse pressure is seen in conditions of high stroke volume such as aortic insufficiency or hyperthyroidism. Narrow pulse pressure is seen with cardiac tamponade. (c) C. Cardiac compression will manifest with distended neck veins and cold clammy skin. (u) D. The onset of diastolic murmur is suggestive of valvular disease, not tamponade. Adults and intravenous drug abusers, which of the following bones is most commonly affected with acute osteomyelitis? A. Femur B. Humerus C. Vertebral spine D. Tibia Explanations (u) A. Long bones are most commonly affected with osteomyelitis in children. (u) B. See A for explanation. (c) C. The bones of the vertebral spine are most commonly affected in a patient with osteomyelitis. Organisms reach the well-perfused vertebral body of adults via spinal arteries and quickly spread from the end plate into the disk space and then to the adjacent vertebral body. The infection may originate in the urinary tract and intravenous drug use carries an increased risk of spinal infection (u) D. See A for explanation 63 year-old female presents with a complaint of chest pressure for one hour, noticed upon awakening. She admits to associated nausea, vomiting, and shortness of breath. 12 lead EKG reveals ST segment elevation in leads II, III, and AVF. Which of the following is the most likely diagnosis? A. Aortic dissection B. Inferior wall myocardial infarction

C. Acute pericarditis D. Pulmonary embolus Explanations (u) A. A patient with aortic dissection will complain of tearing, ripping pain. EKG is often normal, but may reveal left ventricular strain pattern. (c) B. Myocardial infarction often presents with chest pressure and associated nausea and vomiting. ST segment elevation in leads II, III, and AVF are classic findings seen in acute inferior wall myocardial infarction. (u) C. Acute pericarditis presents with atypical chest pain and diffuse ST segment elevation. (u) D. Pulmonary embolism often presents with either no EKG changes or sinus tachycardia. Classically described, rarely seen findings include a large S wave in lead I, a Q wave with T wave inversion in lead III, ST segment depression in lead II, T wave inversion in leads V1-V4 and a transient right bundle branch block A 26 year-old female required 12 units packed red blood cells during a trauma resuscitation and surgical repair of liver and splenic lacerations. The patient is now 6 hours postoperative and has blood oozing from the suture line and IV sites. There is bloody urine in the Foley bag. Laboratory evaluation demonstrates a platelet count of 10,000/microliter, prolonged prothrombin level, and the presence of fibrin split products. Which of the following is the most likely diagnosis? A. acute ABO incompatibility reaction B. disseminated intravascular coagulation C. exacerbation of idiopathic thrombocytopenia D. inadequate repair of the liver lacerations Explanations (u) A. ABO incompatibility results in immediate hemolysis and shock. (c) B. Disseminated intravascular coagulation is characterized by bleeding from many sites as all coagulation factors are consumed and then broken down, leading to decreased fibrinogen level and platelet count, prolonged PT and PTT, and presence of fibrin split products. (u) C. Idiopathic thrombocytopenia is characterized by decreased platelet count, but coagulation factors are normal. (u) D. Bleeding would be localized only and would result in shock if lacerations of the liver were not repaired properly. While awaiting operative removal of pheochromocytoma, which of the following classes of medications are used for control of hypertension? A. alpha-adrenergic blocker B. beta-adrenergic blocker C. ACE inhibitor D. diuretic Explanations (c) A. Alpha-adrenergic blockers are used preoperatively to control hypertension in a patient with pheochromocytoma that occurs from unopposed alpha stimulation when the tumor is manipulated. (u) B. Beta-adrenergic blockers are used to control tachycardia and arrhythmias, if present, after the hypertension has been controlled. (u) C. ACE inhibitors and diuretics have no role in the treatment of hypertension from pheochromocytoma. (u) D. See C for explanation

A 22 year-old female complains of worsening pain, swelling, and tenderness in her left heel for 1 week. She sustained a penetrating injury to the heel two weeks ago when she stepped on a nail while running in tennis shoes. Examination reveals a draining puncture wound with surrounding erythema and exquisite tenderness. X-ray of the left foot demonstrates periosteal reaction associated with the wound. Which organism is classically responsible for this infection? A. Escherichia coli B. Streptococcus pyogenes C. Pseudomonas aeruginosa D. Staphylococcus aureus Explanations (u) A. While gram negative enteric organisms can be a cause of osteomyelitis, E. coli is rarely seen outside of the neonatal period. (u) B. Streptococcus pyogenes is an uncommon (~10 %) cause of acute osteomyelitis. The mechanism of injury suggests a different pathogen. (c) C. Pseudomonas aeruginosa is frequently associated with osteomyelitis involving puncture wounds of the foot. This is believed to result from direct inoculation with P. aeruginosa via the foam padding found in tennis shoes. (u) D. Staphylococcus aureus is the most common infecting organism in cases of acute osteomyelitis, however, the mechanism of injury in this case suggests P. aeruginosa. A 42 year-old male is brought to the emergency department with a stab wound to his right lateral chest wall. On physical examination, the patient is stable with decreased breath sounds on the right with dullness to percussion. An upright chest x-ray reveals the presence of a moderate pleural effusion. Subsequent diagnostic thoracentesis contains bloody aspirate. Which of the following is the next most appropriate intervention? A. Thoracotomy B. Needle aspiration C. Close observation D. Tube thoracostomy Explanations (u) A. A thoracotomy is indicated in a small percentage of patients that do not respond to IV administration of fluids and evacuation of the hemothorax via tube thoracostomy. (u) B. Needle aspiration as treatment for a hemothorax is not recommended as it fails to adequately drain the bloody fluid. (u) C. Close observation is only indicated for small effusions in a patient that remains hemodynamically stable. (c) D. This patient has a hemothorax. Drainage of a hemothorax is best obtained through insertion of a chest tube (tube thoracostomy). On physical examination there is evidence of head trauma. The left side of the chest wall appears to move inward with inspiration and outward with expiration. A chest x-ray reveals multiple rib fractures on the left. Which of the following is the most appropriate intervention? A. Surgical fixation of the fractured ribs B. Application of elastic binders and adhesive tape C. Endotracheal intubation and mechanical ventilation D. Chest physiotherapy that encourages frequent coughing Explanations

(u) A. Surgical fixation of flail chest is less reliable than positive-pressure ventilation and is performed only rarely in the United States. (u) B. While application of elastic binders and adhesive tape was historically utilized to stabile the chest, this intervention has been found to decrease chest expansion and worsen lung atelectasis. (c) C. Indications for early endotracheal intubation and mechanical ventilation in treatment of flail chest include patients that are over the age of 65, have comorbid lung disease and associated severe head trauma. Other indications include shock, three or more associated injuries and fracture of eight or more ribs. (u) D. Conservative treatment for mild to moderate flail chest includes analgesic relief of pain, chest physiotherapy that encourages frequent coughing and restriction of fluids to prevent fluid overload, however this patient needs early ventilatory support A 40 year-old patient with a history of recurrent kidney stones presents with acute onset of right flank pain and hematuria. The patient is afebrile and pain is poorly controlled on oral medications. On CT scan a 1 cm stone is noted in the renal pelvis. Which of the following is the most appropriate intervention for this patient? A. Antibiotics B. Shock wave lithotripsy C. Ureterolithotomy D. Fluid hydration Explanations (u) A. There is no indication of infection and antibiotics are not warranted at this time. (c) B. Extracorporeal shock wave lithotripsy is indicated in patients with stones greater than 6 mm in size or intractable pain. (u) C. Ureterolithotomy is an open surgical procedure and therefore very invasive. In this situation an ESWL would be as effective and safer. (u) D. While fluid hydration is indicated, a stone greater than 6 mm will typically not spontaneously pass and surgical therapy is indicated. Which of the following can optimize quality of life and is an definitive treatment for a patient with refractory heart failure? A. Ventricular assist device B. Intra-aortic balloon counterpulsation C. Cardiac transplantation D. Partial resection of the left ventricle Explanations (u) A. Ventricular assist devices can help to provide temporary circulatory support to those awaiting transplantation. (u) B. IABC is useful in cardiogenic shock, but does not improve quality of life. (c) C. Cardiac transplantation is effective, with survival rates of 80-90% in 1 year, 60-70% over 5 years. It does improve quality of life despite the immunosuppression medications. (u) D. Partial resection of the left ventricle is still experimental and has not shown to have successful results A 22 year-old male presents several hours after sustaining a hand injury when he punched a wall. X-rays of the hand demonstrate fracture of the fifth metacarpal neck with 65 degrees dorsal angulation and a claw hand. What is the most appropriate intervention? A. Antibiotic treatment and ulnar gutter splint immobilization

B. Closed reduction and ulnar gutter splint immobilization C. Open reduction and ulnar gutter splint immobilization D. Ulnar gutter splint immobilization only Explanations (u) A. Antibiotic treatment would be appropriate if the skin was broken and the injury was sustained in a fist fight with the potential for introduced oral flora. The first like antibiotic for human bite is Augmentin. (u) B. Reduction followed by splinting is recommended for fifth metatarsal fractures with angulation of 15-40 degrees. (c) C. Open reduction is indicated with angulation of greater than 40 degrees. (u) D. With angulation of greater than 15 degrees, reduction should be performed prior to splinting. A 60 year-old female injured her right wrist when she slipped and fell onto her outstretched hand. Radiographs show a fracture through the metaphysis of the distal radius with dorsal displacement and angulation. Which of the following splints is the best method of temporary immobilization? A. Dorsal forearm B. Ulnar gutter C. Volar forearm D. Volar with thumb spica Explanations (u) A. The dorsal forearm splint is best used as an alternative to the ulnar or radial gutter splint for protection of fractures of metacarpals two through five. (u) B. Ulnar gutter splints are best for treatment of stable fractures and dislocations of the ulnar wrist and metacarpals. (c) C. The volar forearm splint is best for temporary immobilization of forearm, wrist and hand fractures and is the splint of choice for Colles' fracture. (u) D. A volar splint with thumb spica is used to immobilize the first metacarpophalangeal joint and is useful for scaphoid fractures. A 26 year-old female presents to clinic with a left arm that is swollen and non-tender with bluish discoloration along the upper arm and forearm. She is status post pacemaker insertion in the left upper chest for third degree heart block, one week ago. Pulses are present and the arm is warm, but not red. The pacemaker incision is healing well despite a hematoma and tenderness at the site. Which of the following statements would be appropriate patient education about this condition? A. Reassurance that the discoloration is an expected finding. B. Apply cold compresses to the site of the hematoma. C. Elevation of the involved extremity will increase the swelling. D. Aspirin should be taken to help manage pain. Explanations (c) A. This is indicative of migratory ecchymosis and expected after insertion of a pacemaker. (u) B. Warm compresses will help to decrease the hematoma. (u) C. Elevation will help to decrease the swelling. (u) D. ASA is not the pain medicine of choice in a patient with a hematoma due to its non-clotting properties. An 18 year-old male is involved in a motor vehicle accident with a question of cervical spine fracture. What is the imaging test of choice to initially evaluate this patient and clear his cervical c-spine? A. Positron emission tomography B. Magnetic resonance imaging

C. Computed tomography D. Lateral radiograph Explanations (u) A. There is no role for positron emission tomography in suspected cervical spine injury. (u) B. MRI and CT of the spine may be performed in the setting of acute cervical spine injury when a major fracture or dislocation is identified. (u) C. See B for explanation. (c) D. Cervical spine x-rays are most commonly used as the initial screen for cervical spine injury. A cervical spine series consists of a lateral view, anteroposterior (AP) view, and an odontoid view. The lateral view detects up to 80% of traumatic spine injuries. A 29 year-old male is involved in a motor vehicle crash. On the secondary survey it is noted that there is blood at the meatus and the patient is suspected of having a pelvic fracture. The patient is otherwise stable. Which of the following tests should be done to evaluate the urinary system? A. Voiding cystourethrogram (VCUG) B. Intravenous pyelogram (IVP) C. Urethrogram D. Renal arteriography Explanations (u) A. A VCUG is done to evaluate urinary reflux in children. (u) B. An IVP can be done as part of an evaluation for hematuria however it is rarely used today. (c) C. A urethrogram is the only procedure that should be done to evaluate this type of injury as urethral integrity may have been compromised secondary to the pelvic fracture. Blood at the meatus is the most important finding of suspected urethral injury. (u) D. Renal arteriography does not have any place in this scenario as there is no evidence of a renal injury An 18 year-old male who was struck in the left eye with a baseball presents with ocular pain, periorbital ecchymosis, and restricted upward gaze. Which of the following is the diagnostic study of choice in this patient? A. Zygomatic arch x-ray B. CT scan of the orbit C. Ultrasonography D. Fluorescein staining Explanations (u) A. Zygomatic arch x-ray is not the diagnostic study of choice for a suspected blowout fracture. (c) B. CT scan of the orbit is the study of choice to evaluate a suspected blowout fracture. (u) C. Ultrasound may be helpful if there is a suspected foreign body, however it is not helpful in fracture identification. (u) D. Fluorescein staining is indicated in the evaluation of corneal abrasion, not orbital fracture A 48 year-old female presents to the clinic complaining of hematuria. The patient states that she was found to have hematuria during an insurance physical examination. The patient denies dysuria or frequency. She also denies pain in the abdomen, flank or meatus. She denies any history of previous nephrolithiasis. Urinalysis reveals the urine to be yellow and slightly hazy with a positive dipstick for hemoglobin. Microscopic reveals 5-7 RBCs/HPF without WBCs, bacteria, casts, or crystals. What is the next diagnostic study this patient should undergo? A. CT urography

B. Intravenous pyelogram C. Abdominal ultrasound D. Cystoscopy Explanations (c) A. CT urography with and without contrast should be done to evaluate the upper and lower urinary tract for neoplasms, and benign conditions such as urolithiasis. This has replaced IVP for imaging of the upper tracts. Abdominal ultrasound will not help in this scenario and the role of renal ultrasound in evaluation of hematuria is unclear. Cystoscopy will help to assess the bladder and urethra but will not help with evaluation of the upper urinary tract. (u) B. See A for explanation. (u) C. See A for explanation. (u) D. See A for explanation. Which of the following is first-line treatment for a symptomatic bradyarrhythmia due to sick sinus syndrome? A. Permanent pacemaker B. Radiofrequency ablation C. Antiarrhythmic therapy D. Anticoagulation therapy (c) A. Permanent pacemakers are the therapy of choice in patients with symptomatic bradyarrhythmias in sick sinus syndrome. (u) B. Radiofrequency ablation is used for the treatment of accessory pathways in the heart. (u) C. See A for explanation. (u) D. See A for explanation.

Das könnte Ihnen auch gefallen